Anda di halaman 1dari 69

MyPasTest Online: MRCS A - Jan Exam 2015

01. Anatomy-Head, Neck & Spine (163Qs) 61

01. The inferior sagittal sinus: Single best answer question choose ONE true option only.
Drains directly into the confluence of sinuses YOUR ANSWER
Is found in the falx cerebelli
Is formed within the free, inferior border of the falx cerebri CORRECT ANSWER
Drains into the superior petrosal sinus Page | 1
Contains valves, unlike the other venous dural sinuses.

The inferior sagittal sinus is enclosed in the posterior half or two thirds of the free margin of the
falx cerebri. It is cylindrical in shape. It increases in size as it passes backward and ends in the
straight sinus. It receives several veins from the falx cerebri and occasionally receives a few veins
from the medial surfaces of the hemispheres.

02. You are performing a cranial nerve examination on a patient who has a mild head injury. You
ask them to stick out their tongue. With regards to the examination of the tongue:
It is depressed by the hyoglossus YOUR ANSWER
It is passive during the voluntary phase of swallowing
It is protruded by the styloglossus
It is retracted by the hyoglossus muscle
It receives sensory innervation from the vagus nerve.

The sensory innervation to the tongue is from the VIIth (facial) and IXth (glossopharyngeal) cranial
nerves and the lingual nerve. The tongue deviates to the side of a XIIth cranial nerve lesion on
protrusion, is active during the first stage of swallowing and contains the lingual tonsil in the
dorsum of its posterior third. The tongue is retracted up and back by the styloglossus muscle,
protruded by genioglossus and depressed by the hyoglossus.

03. The total length of the trachea is 10cm. At what level does the trachea begin?
Bifurcation of common carotid artery YOUR ANSWER
Cricoid cartilage CORRECT ANSWER
Hyoid bone
Suprasternal notch
Thyroid cartilage.

The trachea commences just below the cricoid cartilage (at the level of C6). Within the thorax and
on the right, the trachea is in contact with the pleura, vagus and subclavian artery. On its left, the
trachea is in contact with the left recurrent laryngeal nerve, aortic arch and the left common carotid
and subclavian arteries. The trachea ends at the upper border of T5, where it bifurcates.

04. A 72-year-old male presents with a swelling in the anterior triangle of the neck. Biopsy confirms
a superficial submandibular gland carcinoma. Which structure is least likely to be involved? Select
one answer only.
Deep cervical fascia YOUR ANSWER
-------------------------------------------------------------------------------------------------------------------------------------------------------------------------------------------------
-------
Dr Mohammed Shamsul Islam Khan, Medical Officer, Clinical Neuro-Surgery, National Institute of Neuro-Sciences and Hospital
Sher-E-Bangla Nagar, Dhaka-1207, Bangladesh. Mobile: +880 1713 455 662, +880 1685 811979. E-mail: drsikhan@gmail.com
MyPasTest Online: MRCS A - Jan Exam 2015
01. Anatomy-Head, Neck & Spine (163Qs) 61

Platysma
The cervical branch of the facial nerve CORRECT ANSWER
The facial artery
The facial vein.
Page | 2
The submandibular gland is a lobulated gland made up of a superficial and a deep part, which are
continuous with each other around the posterior border of the mylohyoid muscle. Part of the gland
lies infero-laterally, enclosed in an investing layer of deep cervical fascia, platysma muscle and
skin. Laterally it is crossed by the cervical branch of the facial nerve and vein.

The facial artery is related to the posterior and superior aspects of the superficial part of the gland.
The marginal mandibular branch of the facial nerve passes beneath platysma and depressor
angulioris in the proximity of the submandibular gland where it may be damaged during surgery.

05. A 21-year-old male is seen in A&E after an assault outside a night club with pain in his jaw. An
X-ray confirms a fracture of the mandible. Which of the following regions is the weakest part of the
mandible? Select one answer only.
Angle YOUR ANSWER
Canine region CORRECT ANSWER
Coronoid process
Head
Ramus.

Mandibular fractures are commonly the result of blows to the face and it is not uncommon for both
sides to be affected. Fractures of the condyles lead to temporo-mandibular joint dislocation and
usually are as a result of a blow to the chin. The canine region is the weakest part of the bone due
to the length of the root. Angle fractures can run downwards and forwards, which leads to
impaction and so prevents displacement. Fractures of the body always cause laceration to the
buccal mucosa.

06. A 50-year-old man who had suffered long-standingback pain presents to the accident and
emergency department with a sudden deterioration and pain radiating to the lateral malleolus.
What is the most likely cause of these symptoms?
A prolapsed intervertebral disc at the L4/5 interspace YOUR ANSWER
A prolapsed intervertebral disc at he L5/S1 interspace CORRECT ANSWER
A prolapsed intervertebral disc compressing the cauda equina
A prolapsed intervertebral disc compressing the sacral segments of the cord
Collapse of a vertebral body secondary to osteoporosis with subsequent compression of the spinal
canal.

A prolapsed L5S1 disc presses on the S1 spinal nerve (the L5 nerve passes above the prolapsed
disc in the intervertebral foramen and so escapes damage). At the level of prolapse, the spinal
-------------------------------------------------------------------------------------------------------------------------------------------------------------------------------------------------
-------
Dr Mohammed Shamsul Islam Khan, Medical Officer, Clinical Neuro-Surgery, National Institute of Neuro-Sciences and Hospital
Sher-E-Bangla Nagar, Dhaka-1207, Bangladesh. Mobile: +880 1713 455 662, +880 1685 811979. E-mail: drsikhan@gmail.com
MyPasTest Online: MRCS A - Jan Exam 2015
01. Anatomy-Head, Neck & Spine (163Qs) 61

canal contains the cauda equina and not cord per se. The S1 dermatome lies over the lateral
malleolus. Exaggerated reflexes are diagnostic of an upper motor neurone lesion.

The S2 dermatome occupies the posterior aspect of the calf. Osteoporosis can cause painful
collapse of vertebral bodies but rarely causes neurological symptoms and most often affects older
women.
Page | 3
07. You are assisting your neurosurgical consultant performing an anterior cervical discectomy. He
is concerned about injuring the recurrent laryngeal nerve. Which of the following is correct
regarding the right recurrent laryngeal nerve? Single best answer - select one answer only.
Arises from a cranial nerve passing through the foramen magnum YOUR ANSWER
Is related to the inferior thyroid artery CORRECT ANSWER
Passes behind the ligamentum arteriosum
Runs posterior to the oesophagus but anterior to the trachea
Winds around the aortic arch.

The recurrent laryngeal nerves arise from the vagus. On the right, the recurrent laryngeal nerve
winds around the subclavian artery; on the left, it winds around the aortic arch, passing behind the
ligamentum arteriosum. Both nerves run in a groove between the trachea and oesophagus and are
closely related to the inferior thyroid artery.

08. A 63-year-old female undergoes a subtotal thyroidectomy for a toxic multinodular goitre.
Which of the following statements is most likely to be true? Select one answer only.
Damage to the external laryngeal nerve causes loss of low pitched phonation YOUR ANSWER
The inferior thyroid artery arises from the origin of the external carotid artery
The inferior thyroid artery should be ligated as far laterally as possible CORRECT ANSWER
The isthmus lies anterior to the thyroid cartilage
The superior thyroid artery enters the upper pole of the thyroid gland close to the recurrent laryngeal
nerve.

The superior thyroid artery arises from the external carotid artery and enters the upper pole of the
thyroid gland close to the external laryngeal nerve, which supplies the cricothyroid muscle, a
tensor of the vocal cord. Damage to this nerve causes the loss of high-pitched phonation. The
inferior thyroid artery, absent in 5%, arises from the thyrocervical trunk of the subclavian artery.

The inferior thyroid artery should be ligated as far laterally as possible to avoid damaging the
recurrent laryngeal nerve. Damage to one recurrent laryngeal nerve causes a weakened voice,
damage to both causes semiadduction and respiratory difficulties. The isthmus is normally in front
of the second and third tracheal rings, although variations are common.

09. A 22-year-old man sustained facial injuries playing rugby. Which radiological investigation is
the most helpful to plan surgery?
Angiography YOUR ANSWER
Computed tomography (CT) scanning CORRECT ANSWER
-------------------------------------------------------------------------------------------------------------------------------------------------------------------------------------------------
-------
Dr Mohammed Shamsul Islam Khan, Medical Officer, Clinical Neuro-Surgery, National Institute of Neuro-Sciences and Hospital
Sher-E-Bangla Nagar, Dhaka-1207, Bangladesh. Mobile: +880 1713 455 662, +880 1685 811979. E-mail: drsikhan@gmail.com
MyPasTest Online: MRCS A - Jan Exam 2015
01. Anatomy-Head, Neck & Spine (163Qs) 61

Facial view X-rays showing fluid in the paranasal sinuses


MRI
Waters facial view.

Computed tomography (CT) scanning is essential when planning reconstruction of facial fractures, Page | 4
especially complex ones. CT scanning also delineates soft tissue damage adequately. Indirect
signs of a fracture include soft tissue swelling, periorbital or intracranial air and fluid in the
paranasal sinus.

Four facial X-rays are usually taken: Waters view (posterioranterior (PA) with cephalad
angulation), Caldwell (PA view), lateral view and occipito-submentovertex view. Waters view tends
to show all facial structures the best. Lines of Dolan (IIII) are three anatomic contours that
correspond to facially important structures.
10. A 25-year-old male who is in Burns intensive care having sustained 42% burns in a house fire is
on day 12 post injury and remains intubated and ventilated. The Consultant Anaesthetist feels he
will not be ready for extubation for a while and the decision is made to perform a tracheostomy. A
size 7 tracheostomy tube is used. Which of the following does the tube size relate to? Select one
answer only.
Distance from tracheostomy tube tip to carina YOUR ANSWER
Distance of tracheostomy tube from vocal cords
Internal diameter of the tracheostomy tube CORRECT ANSWER
Length of the tracheostomy tube
Outer diameter of the tracheostomy tube.

The first tracheal ring is complete in children and must not be excised for fear of tracheal stenosis.
The preferred method is excision of rings 24. Tracheostomy should be performed if prolonged
intubation is expected; either percutaneous or open approaches are used.

Fenestrated tubes allow air to be diverted superiorly through the vocal cords so speech can be
possible, but this can also be attained if the tube is occluded with a finger. A size 7 tracheostomy
tube relates to an internal diameter (ID) of 7 mm, not the outer diameter or length.

11. The thyroid gland in some cases can have a thyroidea ima artery that supplies the isthmus of
the thyroid. If present, it would take origin: Single best answer question choose ONE true option
only.
From the inferior thyroid artery YOUR ANSWER
Directly from the thyrocervical trunk
From the superior thyroid artery
From the brachiocephalic trunk CORRECT ANSWER
Directly from the external carotid artery.

The thyroidea ima artery, when present, arises from the brachiocephalic trunk (innominate artery)
and ascends in front of the trachea to the lower part of the thyroid gland, which it supplies. It varies
greatly in size and appears to compensate for deficiency or absence of one of the other thyroid
-------------------------------------------------------------------------------------------------------------------------------------------------------------------------------------------------
-------
Dr Mohammed Shamsul Islam Khan, Medical Officer, Clinical Neuro-Surgery, National Institute of Neuro-Sciences and Hospital
Sher-E-Bangla Nagar, Dhaka-1207, Bangladesh. Mobile: +880 1713 455 662, +880 1685 811979. E-mail: drsikhan@gmail.com
MyPasTest Online: MRCS A - Jan Exam 2015
01. Anatomy-Head, Neck & Spine (163Qs) 61

vessels. It occasionally arises from the aorta, the right common carotid, the subclavian or the
internal mammary.

12. You are examining a child in the ENT clinic whose mother says has had previous problems with
his Eustachian tubes but has no other details. The Eustachian tube:
Can be obstructed by an enlarging palatine tonsil YOUR ANSWER Page | 5
Closes during swallowing
Drains the inner ear
Gives attachment to the tensor veli palatini muscle CORRECT ANSWER
Pierces the pharyngobasilar fascia to drain into the oropharynx.

The pharyngotympanic tube drains the middle ear into the nasopharynx by piercing the
pharyngobasilar fascia. The cartilaginous part gives attachment to the tensor veli palatini muscle. It
opens on swallowing under the action of the salpingopharyngeus and tensor palati muscles, and is
obstructed by an enlarging pharyngeal tonsil (adenoids).

13. A 69-year-old diabetic lady with a history of a stroke affecting the right cerebral hemisphere is
referred to the vascular surgeons as a Carotid duplex of the left shows a 75% stenosis of the
internal carotid artery (ICA). Which of the following vessels is the first to arise from the ICA? Select
one answer only.
Anterior cerebral artery YOUR ANSWER
Middle cerebral artery
Ophthalmic artery CORRECT ANSWER
Posterior cerebral artery
Superior thyroid artery.

The ICA does not give rise to any branches within the neck, the superior thyroid artery is usually
the first branch of the external carotid artery.The ophthalmic artery is a branch of the internal
carotid and enters the orbit through the optic foramen. The ophthalmic artery then crosses the
optic nerve to the medial side of the orbit accompanied by the nasociliary nerve.

The supraorbital and nasal arteries are two branches of the ophthalmic artery that supply part of
the skin of the forehead. The ICA bifurcates into the anterior and middle cerebral arteries. The
posterior cerebral artery is formed by the bifurcation of the basilar artery which arises from the
junction of the left and right vertebral arteries.

14. You are inserting a central line into the internal jugular vein. Which of the following is a
tributary of the internal jugular vein?
The external jugular vein YOUR ANSWER
The inferior thyroid vein
The middle thyroid vein CORRECT ANSWER
The retromandibular vein
The transverse cervical vein.
-------------------------------------------------------------------------------------------------------------------------------------------------------------------------------------------------
-------
Dr Mohammed Shamsul Islam Khan, Medical Officer, Clinical Neuro-Surgery, National Institute of Neuro-Sciences and Hospital
Sher-E-Bangla Nagar, Dhaka-1207, Bangladesh. Mobile: +880 1713 455 662, +880 1685 811979. E-mail: drsikhan@gmail.com
MyPasTest Online: MRCS A - Jan Exam 2015
01. Anatomy-Head, Neck & Spine (163Qs) 61

The internal jugular vein receives the facial, pharyngeal, lingual and superior and middle thyroid
veins. It is formed from the sigmoid and inferior petrosal sinus and continues to the
brachiocephalic vein. The external jugular arises from the junction of the posterior auricular vein
and the posterior division of the retromandibular vein and drains into the subclavian. The
transverse cervical vein is a tributary of the external jugular vein.

15. Following a left carotid endarterectomy, a 74-year-old man is found to have altered tongue Page | 6
movements. When asked to protrude his tongue, it deviates to the left. Which of the following
structures could have been affected intra-operatively to cause this?
Ansa cervicalis YOUR ANSWER
Hypoglossal nerve CORRECT ANSWER
Lingual nerve
Marginal mandibular nerve
Vagus nerve.

Damage to the hypoglossal nerve will result in weakness of the extrinsic muscles of the tongue on
the ipsilateral side. The tongue will therefore deviate.

16. A 25-year-old woman with history of high-grade fever, gradual-onset loss of consciousness and
a petechial rash is suspected of having bacterial meningitis. As part of the diagnostic procedure, a
lumbar puncture is to be performed. To avoid injury to the spinal cord and nerves you must insert
the spinal needle just below the spine of the fourth lumbar vertebra. What anatomical landmark
would you use to identify the spine of the fourth lumbar vertebra? Single best answer question
choose ONE true option only.
Pubic symphysis YOUR ANSWER
Iliac crest CORRECT ANSWER
Xiphoid process
Iliac tuberosity
Umbilicus.

The fourth lumbar vertebra (L4) is a relatively safe level for performing a lumbar puncture. Since
the conus medullaris is at the inferior border of L1 or the superior border of L2, it should be safe to
insert a needle either above or below L4. The anatomical landmark used to identify L4 is the top of
the iliac crest.

The line connecting the top of the two iliac crests, the supracrestal line, passes through the
spinous process of the L4 vertebra. Therefore, by finding the tops of the iliac crests, you should be
able to identify L4.

17. A 61-year-old female is commenced on anti-hypertensives by her GP as her blood pressure has
been elevated on the last three visits. Following her first dose at home, she noted her tongue
became very swollen and she had some difficulty breathing. She was rushed to the emergency
department and a diagnosis of angio-neurotic oedema was made. Which group of the following
anti-hypertensives is most likely to have precipitated this? Select one answer only.
ACE inhibitors YOUR ANSWER
-------------------------------------------------------------------------------------------------------------------------------------------------------------------------------------------------
-------
Dr Mohammed Shamsul Islam Khan, Medical Officer, Clinical Neuro-Surgery, National Institute of Neuro-Sciences and Hospital
Sher-E-Bangla Nagar, Dhaka-1207, Bangladesh. Mobile: +880 1713 455 662, +880 1685 811979. E-mail: drsikhan@gmail.com
MyPasTest Online: MRCS A - Jan Exam 2015
01. Anatomy-Head, Neck & Spine (163Qs) 61

blockers
blockers
Calcium channel blockers
Thiazide diuretics.
Page | 7
Angio-neurotic oedema is tongue swelling secondary to ACE inhibitors. The most important initial
management is to secure the airway; a nasotracheal tube may be required. Steroids (which may
take 6 hours to take full effect) and antihistamine should be commenced immediately.

18. An 18-year-old female presented with ear-ache, a conductive deafness and a temperature of
39oC. What is the most common cause?
Pagets disease of bone YOUR ANSWER
Acoustic neuroma
Otosclerosis
A fracture through the petrous temporal bone
Otitis media CORRECT ANSWER.

The most common causes of conductive deafness include wax, acute otitis media, secretory otitis
media, chronic otitis media, barotrauma, otosclerosis and injuries to the tympanic membrane and
otitis externa. Other less common causes include tumours of the middle ear and traumatic
ossicular dislocation.

Sensory neural deafness is caused by a number of causes including infections such as mumps,
herpes zoster, meningitis and syphilis. Other causes include congenitalmaternal rubella,
cytomegalovirus, toxoplasmosis, prolonged exposure to loud noises, drugs (aspirin,
aminoglycosides), Menieres disease, head injury and acoustic neuroma.

Metabolic causes include diabetes and hypothyroidism. In Pagets disease there may be a mixed
hearing loss ie conduction and sensorineural deafness. This is due to direct involvement of the
ossicles of the inner ear due to ankylosis of the stapes, or by impeachment of bone on the eighth
cranial nerve in the auditory foramen.

19. Of the following intrinsic muscles of the larynx, which tenses (stretches) the vocal folds? Single
best answer question choose ONE true option only.
Posterior cricoarytenoid YOUR ANSWER
Lateral cricoarytenoid
Thyroarytenoid
Transverse arytenoid
Cricothyroid muscle CORRECT ANSWER.

-------------------------------------------------------------------------------------------------------------------------------------------------------------------------------------------------
-------
Dr Mohammed Shamsul Islam Khan, Medical Officer, Clinical Neuro-Surgery, National Institute of Neuro-Sciences and Hospital
Sher-E-Bangla Nagar, Dhaka-1207, Bangladesh. Mobile: +880 1713 455 662, +880 1685 811979. E-mail: drsikhan@gmail.com
MyPasTest Online: MRCS A - Jan Exam 2015
01. Anatomy-Head, Neck & Spine (163Qs) 61

The cricothyroid, triangular in form, arises from the front and lateral part of the cricoid cartilage; its
fibres diverge and are arranged in two groups. The lower fibres constitute a pars obliqua and slant
backward and lateralward to the anterior border of the inferior cornu.

The anterior fibres, forming pars recta, run upward, backward and lateralward to the posterior part
of the lower border of the lamina of the thyroid cartilage. The external laryngeal branch of the
superior laryngeal nerve supplies the cricothyroid. The cricothyroids produce tension and Page | 8
elongation of the vocal folds by drawing up the arch of the cricoid cartilage and tilting back the
upper border of its lamina. The distance between the vocal processes and the angle of the thyroid
is so increased and the folds are consequently elongated.

20. You are performing a parotidectomy. Which of the following lie within the gland you are
excising?
External carotid artery YOUR ANSWER
Masseter
Maxillary artery
Superficial temporal artery
Trigeminal nerve.

The parotid gland lies deep to masseter, medial pterygoid, stylomandibular ligament, superficial
temporal artery, maxillary artery and facial nerve. Within the gland lie the facial nerve,
retromandibular vein, external carotid artery and auriculotemporal nerve.

21. A 72-year-old farmer presents with a lesion on his left cheek which has been present for several
months. On examination, it has nodular appearance with rolled edges and visible telangiectasia.
Clinically it is thought to be a basal cell carcinoma (BCC) and the decision is made to proceed with
treatment. What proportion of BCCs occur in the head and neck region? Select one answer only.
1% YOUR ANSWER
10%
30%
50%
90% CORRECT ANSWER.

BCC is the commonest skin tumour on the face and 90% of all BCCs occur in the head and neck
region. Sunlight exposure and genetic factors are the main risk factors.

BCCs rarely, if ever, metastasise and treatment is with complete surgical excision. Radiotherapy is
reserved for recurrences, which are typically aggressive if they recur on the cheeks, nasolabial
folds, medial canso and preauricular region.

The phrase rather a large scar than a small tomb (Sir Harold Gillies) should always be taken into
consideration when planning surgical excision tumours under 1 cm should have at least a 5 mm
excision margin and those over 1 cm should have at least a 1 cm margin.

22. You are assisting your ENT consultant performing a thyroidectomy for malignancy. Regarding
the surgical anatomy of the thyroid gland:
-------------------------------------------------------------------------------------------------------------------------------------------------------------------------------------------------
-------
Dr Mohammed Shamsul Islam Khan, Medical Officer, Clinical Neuro-Surgery, National Institute of Neuro-Sciences and Hospital
Sher-E-Bangla Nagar, Dhaka-1207, Bangladesh. Mobile: +880 1713 455 662, +880 1685 811979. E-mail: drsikhan@gmail.com
MyPasTest Online: MRCS A - Jan Exam 2015
01. Anatomy-Head, Neck & Spine (163Qs) 61

The inferior parathyroid glands are more constant in position than the superior parathyroid glands
YOUR ANSWER
The middle thyroid veins are more constant in position than the superior and inferior thyroid veins
The thyroid gland does not have a definite capsule
There is a ligament, Berrys ligament connecting the thyroid to the cricoid cartilage and upper Page | 9
trachea CORRECT ANSWER
Unilateral recurrent laryngeal nerve division results in the contralateral vocal cord lying in the mid- or
cadaveric position.

The thyroid gland has a definite, fine capsule, which allows a capsular dissection to preserve the
recurrent laryngeal nerves. The superior parathyroid glands are more constant in position than the
inferior.

Because of their embryological migration, the inferior glands may be situated among the
pretracheal lymph nodes or in the thymus as far as 10 cm from the thyroid. The middle thyroid
veins are the least constant of the thyroid veins.

The superior veins drain into the internal jugular vein; the inferior veins are very constant and drain
into the brachiocephalic veins; and the middle veins are very variable and often multiple. Unilateral
recurrent laryngeal nerve section results in the ipsilateral vocal cord lying motionless in the mid- or
cadaveric position.

The voice is hoarse and weak. If both recurrent laryngeal nerves are divided, then the glottic space
is narrowed and stridor develops.

23. There are three main foramina: the superior and inferior orbital fissures, and the optic canal.
Which of the following enter the orbit via the optic canal?
Abducens nerve YOUR ANSWER
Lacrimal nerve
Ophthalmic artery CORRECT ANSWER
Superior and Inferior ophthalmic veins
Trochlear nerve.

The optic nerve and ophthalmic artery pass through the optic canal. The optic nerve and retina are
part of the central nervous system. The optic nerve is invested by all the meningeal layers.

Since the central retinal artery travels in the optic nerve after branching off the ophthalmic artery,
damage to the optic nerve commonly causes retinal infarction. The other options in this question
pass through the superior orbital fissure.

24. A 15-year-old rugby player sustains a head injury during a match and collapses after initially
playing on. He is taken to A+E where a CT scan demonstrates a large left extradural haematoma. At
operation, damage to the middle meningeal artery is noted to be the cause. Which of the following
is correct regarding this artery? Select one answer only.

-------------------------------------------------------------------------------------------------------------------------------------------------------------------------------------------------
-------
Dr Mohammed Shamsul Islam Khan, Medical Officer, Clinical Neuro-Surgery, National Institute of Neuro-Sciences and Hospital
Sher-E-Bangla Nagar, Dhaka-1207, Bangladesh. Mobile: +880 1713 455 662, +880 1685 811979. E-mail: drsikhan@gmail.com
MyPasTest Online: MRCS A - Jan Exam 2015
01. Anatomy-Head, Neck & Spine (163Qs) 61

Arises from the superficial temporal branch of the external carotid artery YOUR ANSWER
Enters the skull through the foramen ovale
Is unlikely to be damaged in fractures at the pterion
Originates in the pterygopalatine fossa
Supplies part of the dura mater and the calvaria CORRECT ANSWER.
Page |
10
The middle meningeal artery divides from the maxillary branch of the external carotid, entering the
cranium through the foramen spinosum. It is the largest artery that supplies the dura and also the
calvaria. While the maxillary artery does give off branches in the pterygopalatine fossa region, the
middle meningeal artery comes off more proximally, deep to the ramus of the mandible.

It is particularly prone to damage caused by temporal bone fractures. The anterior division runs
beneath the pterion, which is the junction of the parietal, frontal and squamous temporal bones
with the greater wing of the sphenoid, and is prone to damage from fractures here.

25. Which of the following types of nerve fibre does the spinothalamic tract carry?
Descending motor fibres YOUR ANSWER
First order fibres carrying light touch and proprioception
First order fibres carrying pain and temperature sensation
Second order fibres carrying light touch and proprioception
Second order fibres carrying pain and temperature sensation CORRECT ANSWER.

The spinothalamic tract conveys pain, temperature, touch and pressure sensations from one side
of the body to the opposite side of the brain. Vibration and position sense are conveyed via the
posterior column. The first neurone of the spinothalamic tract synapses in the posterior horn; the
next neurone crosses to the right side of the spinal cord and synapse in the thalamus, after
ascending through the cord and brainstem; the third neurone arises in the thalamus to pass to the
cortex.

The secondary axons of the spinothalamic tract ascend through the brainstem to synapse in the
thalamus. Axons from the cervical region synapse medially while axons from the lumbar region
synapse laterally. A lesion of the spinothalamic tract anywhere in the brainstem would lead to a
loss of pain sensations from the opposite side of the body.

Temperature and touch sensations would also be diminished from the opposite side of the body
but not totally lost because other pathways may also convey these modalities. A lesion of the
spinothalamic tract at the level of the spinal cord would lead to loss of pain sensations on the
opposite side, beginning one level below the level of the lesion.

26. What is the function of the middle nasal meatus?


It contains the bulla ethmoidalis of the middle ethmoidal air sinus YOUR ANSWER
-------------------------------------------------------------------------------------------------------------------------------------------------------------------------------------------------
-------
Dr Mohammed Shamsul Islam Khan, Medical Officer, Clinical Neuro-Surgery, National Institute of Neuro-Sciences and Hospital
Sher-E-Bangla Nagar, Dhaka-1207, Bangladesh. Mobile: +880 1713 455 662, +880 1685 811979. E-mail: drsikhan@gmail.com
MyPasTest Online: MRCS A - Jan Exam 2015
01. Anatomy-Head, Neck & Spine (163Qs) 61

It drains the nasolacrimal duct


It drains the posterior ethmoidal air sinus
It drains the sphenoidal air cells
It is lined by olfactory epithelium containing the primary olfactory neurons.
Page |
The nasolacrimal duct drains into the inferior meatus; the sphenoidal air sinus drains into the
sphenoethmoidal recess; and the posterior ethmoidal air sinus drains into the superior meatus.
11
The middle meatus contains the bulla ethmoidalis of the middle ethmoidal air sinus, which drains
through a hiatus in the bulla. Olfactory epithelium containing the primary olfactory neurones lines
the superior nasal recess (roof of the nasal cavity) under cover of the cribriform plate of the
ethmoid bone.

27. A 24-year-old rugby player was injured in the face during a match. Eye injuries are common in
sports. Which complication results from fractures of the orbital floor ('blowout fractures')?
Damage to the mental nerve leading to cheek numbness YOUR ANSWER
Diplopia especially on upward gaze CORRECT ANSWER
Hyphema
Ocular muscles rupture
Retinal detachment.

Blowout fractures occur along the orbital floor, as this is the thinnest part of the bone. Structures
may herniate through into the ethmoidal or maxillary sinuses, causing the trapdoor appearance
on plain X-ray. Ocular injury occurs in 24% of cases, with enophthalmos and diplopia the
commonest signs. Computed tomography (CT) helps to delineate the fracture and aids in planning
surgery if required. Damage to the infraorbital nerve leads to cheek numbness.

28. You are reviewing a patient in the ENT clinic with a painful swelling around the jaw and you
suspect there is a mass in the submandibular salivary gland. The submandibular gland:
Has the facial artery running through it YOUR ANSWER (Correct)
Has the hypoglossal nerve running through it
Is deep to the hyoglossus muscle
Lies entirely below the lower mandible
Lies below the digastric muscle.

The submandibular gland consists of a deep and a superficial part. The superficial part lies in the
digastric triangle (above and between the two bellies of the digastric muscle). The hypoglossal
nerve runs medial to the superficial part of the gland. The gland is superficial to the mylohyoid and
hyoglossus muscles. A third of the submandibular gland lies below the lower border of the
mandible and two-thirds above it.

29. Itching sensation from the skin immediately over the base of the spine of your scapula is
mediated through the: Single best answer question choose ONE true option only.
-------------------------------------------------------------------------------------------------------------------------------------------------------------------------------------------------
-------
Dr Mohammed Shamsul Islam Khan, Medical Officer, Clinical Neuro-Surgery, National Institute of Neuro-Sciences and Hospital
Sher-E-Bangla Nagar, Dhaka-1207, Bangladesh. Mobile: +880 1713 455 662, +880 1685 811979. E-mail: drsikhan@gmail.com
MyPasTest Online: MRCS A - Jan Exam 2015
01. Anatomy-Head, Neck & Spine (163Qs) 61

Accessory nerve YOUR ANSWER


Dorsal primary ramus of T2 CORRECT ANSWER
Dorsal root of T2
Ventral primary ramus of T2
Ventral root of T2.
Page |
12
Dorsal and ventral primary rami are the first branches off spinal nerves. Dorsal rami provide
sensory innervation to the skin over the back and give motor innervation to the true back muscles;
ventral rami supply sensory innervation to the skin over the limbs and the skin over the ventral
side of the trunk.

Ventral rami also give motor innervation to the skeletal muscles of the neck, trunk and extremities.
The accessory nerve, which innervates the trapezius, is not responsible for any sensory
innervation. The dorsal and ventral roots of spinal nerves are not directly responsible for any
sensory innervation to the skin.

Dorsal and ventral rootlets emerge from the spinal cord to form the dorsal and ventral roots. The
ventral roots contain efferent motor fibres to skeletal muscles, while the dorsal roots contain
afferent sensory fibres. These roots combine to form the spinal nerve, which then gives off the
primary rami.

30. A 7-year-old boy is seen by his GP for otitis media of his left ear, on further questioning it
emerges he has had recurrent upper respiratory tract infections and has erythematous swollen
tonsils at present. Which part of the skull does the Eustachian tube penetrate? Select one answer
only.
Greater wing of the sphenoid YOUR ANSWER
Lesser wing of the sphenoid
Occipital bone
Petrous temporal bone CORRECT ANSWER
Squamous temporal bone.

The Eustachian (Phanyngotympanic, auditory) tube connects the tympanic cavity to the
nasopharynx. The posterolateral third is bony and the remainder cartilaginous.The Eustachian tube
in a child is shorter and more horizontal. The opening of the auditory tube lies above the soft palate
adjacent to the tubal tonsil. The bony part of the Eustachian tube perforates the petrous temporal
bone.

31. You are in theatre performing a formal tracheostomy. When performing this procedure:
It is best performed with the neck slightly flexed YOUR ANSWER
Midline dissection provides a relatively avascular plane CORRECT ANSWER
Once the tracheostomy cuff is inflated further fixation is not required
The first and second tracheal rings are incised

-------------------------------------------------------------------------------------------------------------------------------------------------------------------------------------------------
-------
Dr Mohammed Shamsul Islam Khan, Medical Officer, Clinical Neuro-Surgery, National Institute of Neuro-Sciences and Hospital
Sher-E-Bangla Nagar, Dhaka-1207, Bangladesh. Mobile: +880 1713 455 662, +880 1685 811979. E-mail: drsikhan@gmail.com
MyPasTest Online: MRCS A - Jan Exam 2015
01. Anatomy-Head, Neck & Spine (163Qs) 61

The skin incision is placed between the cricoid cartilage and hyoid.

Tracheostomy is a definitive surgical airway. The head and neck are extended, and a 34 cm
transverse collar incision is made between the cricoid cartilage and sternal notch. Once the
incision is deepened through the platysma muscle, the remainder of dissection should continue in
the midline in a relatively avascular plane down to the pretracheal fascia. Page |
This layer is then incised to reveal the tracheal rings. Care must be taken not to incise the first ring 13
(the cricoid cartilage). A vertical incision through the second and third rings is usual. Once the
airway is in the trachea, the cuff may be inflated. Further fixation is then mandatory to prevent the
tracheostomy tube from being displaced.

32. Which of the following structures is least likely to be encountered on left carotid
endarterectomy?
Facial vein YOUR ANSWER
Pleural membranes CORRECT ANSWER
Sympathetic chain
Thoracic duct
Vagus nerve.

The vagus nerve travels in the carotid sheath in the neck. The sympathetic trunk lies alongside the
cervical vertebrae, immediately behind the carotid artery, and has three cervical ganglia (superior,
middle and inferior). During a left carotid endarterectomy (CEA), the thoracic duct may be seen.

Pleural membranes are usually a little deeper and lateral to the site of CEA. The hypoglossal nerve
passes forwards to supply the tongue and has to be protected, while the facial vein has to be
ligated and divided to mobilise the internal jugular vein.

33. A patient undergoes excision of the left submandibular salivary gland for sialectasia.
Unfortunately, his hypoglossal (XII) nerve on that side is damaged. What is the most likely
outcome? Single best answer question choose ONE true option only.
There is numbness of the posterior one-third of the tongue YOUR ANSWER
On protruding the tongue, it deviates towards the right
The uvula deviates towards the left
All the intrinsic muscles of the left side of the tongue are paralysed CORRECT ANSWER
The genioglossus muscle is spared.

The hypoglossal nerve supplies all the muscles of the tongue but none of the palate (the
palatoglossus muscle, supplied by the vagus nerve, is a muscle of the palate). It has no sensory
component. The genioglossus muscle protrudes the tongue; when it is paralysed, the muscle on
the opposite side is unaffected and deviates the tongue towards the affected side.

-------------------------------------------------------------------------------------------------------------------------------------------------------------------------------------------------
-------
Dr Mohammed Shamsul Islam Khan, Medical Officer, Clinical Neuro-Surgery, National Institute of Neuro-Sciences and Hospital
Sher-E-Bangla Nagar, Dhaka-1207, Bangladesh. Mobile: +880 1713 455 662, +880 1685 811979. E-mail: drsikhan@gmail.com
MyPasTest Online: MRCS A - Jan Exam 2015
01. Anatomy-Head, Neck & Spine (163Qs) 61

34. An 82-year-old woman fainted and fell downstairs sustaining head injuries with skull fracture.
Bleeding was discovered from the middle meningeal artery. What is this is associated with?
Injury to the posterior branch of maxillary artery YOUR ANSWER
Extradural haematoma CORRECT ANSWER
Shifting of the brain causing ipsilateral pupillary constriction Page |
Basal skull fracture
14
Decrease in the intracranial pressure.

The anterior branch is more often affected than the posterior, probably because it courses through
thinner bone and across the sutures at the pterion. Although designated an extradural bleed, the
haematoma collects between the endosteal and meningeal layers of the dura mater. Fractures
invariably precede an extradural bleed. The pia mater invests the blood vessel, not the arachnoid
mater.

35. You are treating a patient who is going to undergo parotidectomy and he asks you about
parotid neoplasms. Which of the following is correct?
Computed tomography (CT) rather than magnetic resonance imaging (MRI) is the radiological
investigation of choice for staging of parotid carcinomas YOUR ANSWER
Presentation with facial nerve palsy implies malignancy CORRECT ANSWER
They are commonly bilateral
They are malignant in 50%
They usually present with maxillary numbness.

Parotid neoplasms are usually unilateral, only 1520% of parotid tumours are malignant. The main
mode of presentation is a symptomless swelling, often dating back several years. Presentation
with facial nerve palsy is very suggestive of a malignancy.

Rare presentations include trismus and referred pain via the trigeminal nerve. MRI is used for
radiological staging as there is better soft tissue discrimination, imaging can be carried out in
multiple planes and it is easier to detect cervical lymphadenopathy.

36. The olfactory foramina: Single best answer question choose ONE true option only.
Are located in the middle cranial fossa YOUR ANSWER
Are located in the anterior cranial fossa CORRECT ANSWER
Are located immediately inferior to the optic foramen
Have motor neurones running through them
Are located in the sphenoid bone.

The olfactory foramina are located in the anterior cranial fossa. These foramina are in the
cribriform plate of the ethmoid bone for the passage of olfactory nerves.

-------------------------------------------------------------------------------------------------------------------------------------------------------------------------------------------------
-------
Dr Mohammed Shamsul Islam Khan, Medical Officer, Clinical Neuro-Surgery, National Institute of Neuro-Sciences and Hospital
Sher-E-Bangla Nagar, Dhaka-1207, Bangladesh. Mobile: +880 1713 455 662, +880 1685 811979. E-mail: drsikhan@gmail.com
MyPasTest Online: MRCS A - Jan Exam 2015
01. Anatomy-Head, Neck & Spine (163Qs) 61

37. A 24-year-old male is rushed into A&E after being kicked in the neck during a karate
competition. He has a hoarse voice and is very short of breath. On examination, there is a marked
stutter, laryngeal tenderness, subcutaneous emphysema and the patient is drooling saliva. What is
the first line treatment?
Nasopharyngeal tube insertion YOUR ANSWER
CT angiogram of neck Page |
Emergency tracheostomy 15
Endotracheal intubation with C-spine immobilisation CORRECT ANSWER
Surgical cricothyroidotomy.

Fracture of the larynx can present with acute airway obstruction. The mechanism of injury usually
involves either blunt or penetrating trauma to the larynx. Although the larynx is protected by the
mandible above and the sternum below, it may be crushed, between these two points, by a blunt
object anteriorly and the cervical spine posteriorly.

It is diagnosed by the following triad: hoarseness of voice, subcutaneous emphysema and a


palpable fracture. Other signs and symptoms include stridor, dysphagia, haemoptysis, laryngeal
tenderness, odynophagia and anterior neck pain. Furthermore, there may be loss of thyroid
cartilage prominence and ecchymosis in the overlying skin.

If the patients airway is totally obstructed or the patient is in severe respiratory distress, an
attempt at endotracheal intubation is warranted (note that in trauma, C-spine immobilisation should
be performed together with securing the airway). Flexible endoscopic-guided intubation may be
helpful in this situation, but only if it can be performed promptly and by an experienced
anaesthetist or surgeon. Surgical cricothyroidotomy, although not preferred, may be a lifesaving
option.

If intubation is unsuccessful and if surgical cricothyroidotomy cannot be performed, then an


emergency tracheostomy is indicated. However, in emergency conditions, tracheostomy is difficult
to perform, may be associated with profound bleeding and may be time consuming.

Computed tomography is the investigation of choice in suspected laryngeal trauma. It is especially


useful in evaluating the extent of injury when flexible fibre optic examination is limited by laryngeal
oedema.

38. A 20-year-old girl presented with a unilateral conductive hearing loss; otoscopy revealed
characteristics of Cholesteatoma of the middle ear. What would be the most appropriate
management plan?
As it is a benign tumour composed of cholesterol it should be left alone YOUR ANSWER
It should be completely removed at initial surgery CORRECT ANSWER
It could be left as it does not pose a risk of meningitis
As it usually presents with a neck abscess it should be treated with incision and drainage
Radiotherapy is appropriate and effective as it is composed of atypical cells.

-------------------------------------------------------------------------------------------------------------------------------------------------------------------------------------------------
-------
Dr Mohammed Shamsul Islam Khan, Medical Officer, Clinical Neuro-Surgery, National Institute of Neuro-Sciences and Hospital
Sher-E-Bangla Nagar, Dhaka-1207, Bangladesh. Mobile: +880 1713 455 662, +880 1685 811979. E-mail: drsikhan@gmail.com
MyPasTest Online: MRCS A - Jan Exam 2015
01. Anatomy-Head, Neck & Spine (163Qs) 61

Cholesteatoma derived its name from early observations that it resembled a ball of cholesterol.
Although given the suffix oma, it does not behave like a tumour, nor is it composed of atypical
cells suggestive of a tumour.

It is in fact a non-cleaning squamous cell cyst that can cause complications if untreated, due to its
invasive properties, including meningitis, brain abscess, hearing loss, neck abscess (Bezolds Page |
abscess, very rare), lateral sinus thrombosis, facial nerve palsy and vertigo.
16
It usually presents with continuously discharging ears and otalgia. Treatment is regular cleaning
and exteriorisation in the form of mastoidectomy and variations of this procedure.

39. A 35-year-old man presents to hospital with progressive deterioration of his vision over a 3-
month period. His left visual acuity was 5/20. Enhanced orbital computed tomographic (CT) scans
revealed a dilatation of the ophthalmic artery. What is the next most appropriate step in his
management?
Cerebral angiography of the internal carotid artery to assess the anatomy of the aneurysm YOUR
ANSWER
Immediate surgery to clip the aneurysm to prevent further deterioration in vision
Immediate treatment with high dose corticosteroids is indicated because this presentation is most
likely to be secondary to giant cell arteritis
No immediate action in required. This patient should be followed up with serial orbital CT scans every
3 months
Selective cerebral angiography of the external carotid artery to assess the anatomy of the aneurysm.

The ophthalmic artery is a branch of the internal carotid artery (ICA); it passes through the optic
canal and gives off anterior and posterior ethmoidal branches and the supra-orbital branch to the
forehead.

Ophthalmic artery aneurysms usually occur on the carotid-ophthalmic junction. True ophthalmic
artery aneurysms are rare. Whilst the chance of rupture is low the proximity of such aneurysms to
the optic nerve as they pass through the optic canal often causes deterioration in vision.

Visual symptoms respond poorly to surgical treatment. In this case the next step is to assess the
aneurysm using angiography. The size, shape and location of the aneurysm a decision will
determine whether an open or endovascular approach should be used. The rapid deterioration in
the patients symptoms warrants prompt further investigation.

Giant cell arteritis is an inflammatory disease of the blood vessels, which mainly affects branches
of the external carotid artery. It is more common in men (2:1) with a mean age of onset >55 years.
People present with; bruits, fever, headache, jaw and tongue claudication, tenderness over the
scalp and visual disturbance. Treatment is with high dose corticosteroids.

-------------------------------------------------------------------------------------------------------------------------------------------------------------------------------------------------
-------
Dr Mohammed Shamsul Islam Khan, Medical Officer, Clinical Neuro-Surgery, National Institute of Neuro-Sciences and Hospital
Sher-E-Bangla Nagar, Dhaka-1207, Bangladesh. Mobile: +880 1713 455 662, +880 1685 811979. E-mail: drsikhan@gmail.com
MyPasTest Online: MRCS A - Jan Exam 2015
01. Anatomy-Head, Neck & Spine (163Qs) 61

40. You are assisting in a carotid endarterectomy and are asked about the anatomy of the internal
carotid artery. Which statement is correct regarding the internal carotid artery?
It commences at the level of C6 YOUR ANSWER
It divides into the middle and posterior cerebral arteries
It gives off the ophthalmic artery CORRECT ANSWER Page |
It has two extra-cranial branches
17
It passes through the foramen ovale.

The common carotid artery bifurcates into the external and internal carotids at the level of the
upper part of the C4 vertebra ,ie the upper border of the thyroid cartilage, however this bifurcation
is frequently higher, near the tip of the great horn of the hyoid bone (C3 level).
The internal carotid artery has no extra cranial branches and enters the base of the skull in the
petrous temporal bone through the carotid canal.

The internal carotid on entering the skull passes forwards through the temporal bone upwards into
the cavernous sinus, turns forward and upwards through the roof of the sinus to lie medial to the
anterior clinoid process before turning back on itself above the cavernous sinus and then passing
once more lateral to the optic chiasma to end by dividing into the anterior and middle cerebral
arteries. The ophthalmic artery originates from the internal carotid artery immediately above the
roof of the cavernous sinus.

41. A 54-year-old female undergoes a wide local excision of a basal cell carcinoma to her left
nasolabial region. It is decided to proceed with a full-thickness skin graft to replace the defect.
Which of the following sites would be the best donor site for this? Select one answer only.
Anterior forearm YOUR ANSWER
Anterior thigh
Inguinal region
Postauricular area CORRECT ANSWER
Posterior forearm.

Although the forearm and thigh are good donor sites for split thickness skin grafting, they have
limitations for facial reconstruction in comparison with other sites. Pre- and post-auricular areas
are excellent for facial reconstruction as they are hairless and usually match skin colour and
texture. Often several stages of reconstruction are required to obtain the best possible cosmetic
result. If possible, facial soft tissue should be used to replace facial defects, hence forehead flaps
are suitable for nasal reconstructions.

In this case, wide local excision of the malignant lesion would necessitate a full-thickness skin
graft. Such grafts include all skin layers down to the level of the dermal fat, and the donor defect
must be primarily closed as no skin element remains in the donor bed. The size and site of such
grafts are therefore limited to areas where there are sufficient skin laxity to allow for primary
closure (e.g. inguinal or wrist creases, and post-auricular regions). Full thickness grafts also tend
to retain their colour and texture, and do not contract significantly.

-------------------------------------------------------------------------------------------------------------------------------------------------------------------------------------------------
-------
Dr Mohammed Shamsul Islam Khan, Medical Officer, Clinical Neuro-Surgery, National Institute of Neuro-Sciences and Hospital
Sher-E-Bangla Nagar, Dhaka-1207, Bangladesh. Mobile: +880 1713 455 662, +880 1685 811979. E-mail: drsikhan@gmail.com
MyPasTest Online: MRCS A - Jan Exam 2015
01. Anatomy-Head, Neck & Spine (163Qs) 61

42. A 78-year-old male is about to undergo a left Carotid endarterectomy, the anaesthetist has
discussed the anaesthetic options with the patient and they have decided to attempt a Cervical
plexus block. Which of the following nerves lie superficial to the sternocleidomastoid muscle and
may be affected by this block? Select one answer only.
Greater occipital nerve YOUR ANSWER
Hypoglossal nerve Page |
Phrenic nerve 18
Suprascapular nerve
Traverse cervical nerve CORRECT ANSWER.

The Cervical plexus is located in the posterior triangle and is formed by union of the anterior rami
of the first four cervical nerves. The superficial branches of the cervical plexus are the lesser
occipital nerve, greater auricular nerve, transverse cervical nerve and supraclavicular nerve.

Local anaesthetic can be injected along the posterior border of the sternocleidomastoid (between
the superior and inferior thirds) the nerve point of the neck for a cervical plexus block. The
transverse cervical nerve emerges as a single trunk behind the posterior border of the
sternocleidomastoid and is superficial to the muscle.

43. A 27-year-old male is seen in A&E after being admitted following an alleged assault on a night
out. He is very inebriated, but examination reveals malocclusion of the teeth and an x-ray shows a
displaced fracture of the mandible. It is thought he will require surgery, which of the following
imaging modalities is best to plan reconstruction? Select one answer only.
CT YOUR ANSWER (Correct)
Fluoroscopy
Lateral X-rays
MRI
Occipitomental view (OM) X-ray.

Mandibular fractures are among the commonest facial fracture, given its prominence on the face.
Fractures usually obey the ring bone rule, meaning that a fracture at one point usually indicates a
corresponding fracture elsewhere. Approximately 50% of mandibular fractures are bilateral.
Therefore multiple fractures are the norm.

Malocclusion of the teeth commonly occurs, temporo-mandibular joint (TMJ) dysfunction occurs
with condylar fractures. Computed tomography (CT) scanning is the imaging of choice after plain
X-rays in planning reconstruction.

44. You see a patient in clinic with a malignant thyroid swelling. Which one of the following is
correct regarding the types of thyroid cancer?
Anaplastic cancer has a good prognosis YOUR ANSWER
Anaplastic cancer is commoner in younger patients
Follicular carcinoma cannot be diagnosed on fine needle aspiration (FNA) CORRECT ANSWER
Follicular carcinoma is the most common type
-------------------------------------------------------------------------------------------------------------------------------------------------------------------------------------------------
-------
Dr Mohammed Shamsul Islam Khan, Medical Officer, Clinical Neuro-Surgery, National Institute of Neuro-Sciences and Hospital
Sher-E-Bangla Nagar, Dhaka-1207, Bangladesh. Mobile: +880 1713 455 662, +880 1685 811979. E-mail: drsikhan@gmail.com
MyPasTest Online: MRCS A - Jan Exam 2015
01. Anatomy-Head, Neck & Spine (163Qs) 61

Medullary cancer only occurs in patients with multiple endocrine neoplasia type II (MEN II).

Anaplastic cancer has a very poor prognosis, and is usually seen in older patients.
Hypoparathyroidism is a well recognised complication of surgery. Follicular carcinoma cannot be
differentiated from follicular adenoma by FNA.
Page |
Medullary cancer of the thyroid is associated with the MEN II syndrome but most patients who have
medullary cancer will not have MEN. Female to male ratio for thyroid cancer is 3 : 1. Papillary 19
adenocarcinoma is the commonest type (70%).

45. Which of the following effects is most likely to be seen if the right dorsal scapular nerve is
injured near its origin? Single best answer question choose ONE true option only.
Skin of the upper back on the right side would be numb YOUR ANSWER
The point of the right shoulder would droop
Scapular retraction on the right would be weakened CORRECT ANSWER
Extension of the right arm would be weakened
Inability to adduct the right arm.

The dorsal scapular nerve is a motor nerve off the C5 nerve root that innervates the rhomboids and
levator scapulas. These muscles help to retract and elevate the scapula, so these movements
would be weakened following that damage. The skin of the upper back on the right side is
innervated by the dorsal primary rami of a spinal nerve.

The point of the right shoulder, the acromion, is elevated by the trapezius. The trapezius is
innervated by the accessory nerve, so the point of the shoulder would droop if the accessory nerve
was damaged. Latissimus dorsi, innervated by the thoracodorsal nerve, allows for extension and
adduction of the arm.

46. A 14-year-old boy presents to the Accident and Emergency department with erythematous,
swollen tonsils and left ear pain. Examination of the ear is unremarkable. Referred pain from which
nerve is most likely to be responsible for these symptoms? Select one answer only.
Facial nerve YOUR ANSWER
Glossopharyngeal nerve CORRECT ANSWER
Hypoglossal nerve
Lesser palatine nerve
Superior laryngeal nerve.

Earache from acute tonsillitis is due to referred pain along the glossopharyngeal nerve, which runs
in the tonsillar bed and sends a tympanic branch to the tympanic plexus in the middle ear. The
other nerves are not involved.

47. Endocrine abnormalities can be associated with neurosurgical conditions. The hypthalmus and
pituitary gland are the dominant controllers of the endocrine system. What structure lies lateral to
the pituitary gland?

-------------------------------------------------------------------------------------------------------------------------------------------------------------------------------------------------
-------
Dr Mohammed Shamsul Islam Khan, Medical Officer, Clinical Neuro-Surgery, National Institute of Neuro-Sciences and Hospital
Sher-E-Bangla Nagar, Dhaka-1207, Bangladesh. Mobile: +880 1713 455 662, +880 1685 811979. E-mail: drsikhan@gmail.com
MyPasTest Online: MRCS A - Jan Exam 2015
01. Anatomy-Head, Neck & Spine (163Qs) 61

Cavernous sinus YOUR ANSWER (Correct)


Diaphragma sellae
Optic chiasm
Sphenoid sinus
Temporal lobe.
Page |
20
The pituitary sits in the sella turcica of the sphenoid bone. It is covered by the diaphragma sellae
(fold of dura) which separates it from the optic chiasma above. There is an opening in the
diaphragma for the pituitary stalk. The cavernous sinuses are found laterally including the internal
carotid artery and cranial nerves III, IV, Va, Vb and VI and the sphenoid sinuses inferiorly.

48. You are in the ENT clinic reviewing a patient with a midline swelling of the neck. The differential
diagnosis includes:
Arteriovenous fistula YOUR ANSWER
Branchial cyst
Chemodectoma
Cystic hygroma
Subhyoid bursa CORRECT ANSWER

Swellings of the neck in the midline include thyroglossal cysts, pharyngeal pouches, plunging
ranulae, subhyoid bursae, laryngoceles and lesions in the thyroid isthmus.

49. A 22-year-old female student presents with epistaxis. This fails to improve with pressure. Which
of the following methods would be attempted next?
Embolisation YOUR ANSWER
External carotid artery ligation
Sphenopalatine artery ligation
Trans-sphenoidal ligation
Use of silver nitrate CORRECT ANSWER.
Silver nitrate or chemical cautery is often the first method tried and can be very effective. Ligation
of local (sphenopalatine) and, in severe life-threatening bleeds, regional (external carotid) arteries
are not uncommonly seen. Maxillary artery embolisation is becoming a more frequently used
alternative.

50. The name of the bone that the internal carotid artery enters to reach the intracranial cavity is
the: Single best answer question choose ONE true option only.
Frontal YOUR ANSWER
Occipital
Sphenoid
Temporal CORRECT ANSWER

-------------------------------------------------------------------------------------------------------------------------------------------------------------------------------------------------
-------
Dr Mohammed Shamsul Islam Khan, Medical Officer, Clinical Neuro-Surgery, National Institute of Neuro-Sciences and Hospital
Sher-E-Bangla Nagar, Dhaka-1207, Bangladesh. Mobile: +880 1713 455 662, +880 1685 811979. E-mail: drsikhan@gmail.com
MyPasTest Online: MRCS A - Jan Exam 2015
01. Anatomy-Head, Neck & Spine (163Qs) 61

Maxillary.

The internal carotid artery supplies the anterior part of the brain, the eye and its appendages and
the forehead and nose. In the adult it is the same size as the external carotid, but in the child it is
larger than the external carotid. It is remarkable for the number of curvatures that it presents in
different parts of its course. Page |
It occasionally has one or two curvatures near the base of the skull, while in its passage through 21
the carotid canal in the petrous part of the temporal bone and along the side of the body of the
sphenoid bone it describes a double curvature and resembles an italic letter s. The carotid canal
is found on the inferior surface of the petrous part of the temporal bone.

It ascends vertically at first and then bends and runs horizontally forward and medially. It transmits
the internal carotid artery and the carotid plexus of nerves into the cranium.

51. You are in the ENT clinic reviewing a patient with chronic infection of the middle ear, otitis
media. The middle ear:
Contains all the auditory ossicles YOUR ANSWER
Has a promontory in the projection of the lateral semicircular canal
Has the facial nerve running through its roof
Has the internal carotid artery running medially and posteriorly
Has the internal jugular vein running medially and superiorly.

The medial wall, which separates the middle from the internal ear, contains the oval window and
promontory. The promontory is the rounded projection of the first turn of the cochlea. Also
medially, the internal auditory meatus carries the facial nerve. The floor is a thin bone separating
the middle ear from the bulb of the jugular vein. Anteriorly, a thin bony wall separates the cavity
from the internal carotid artery.

52. A patient has her inferior laryngeal nerve inadvertently divided during a partial thyroidectomy.
Which clinical features are likely to result from this? Single best answer question choose ONE
true option only.
The larynx is anaesthetised inferior to the vocal cord on the affected side YOUR ANSWER
The larynx is totally anaesthetised on the affected side
All the laryngeal muscles on the affected side are paralysed
All the laryngeal muscles are paralysed on the affected side, apart from the posterior cricoarytenoid
muscle
At laryngoscopy, the affected cord is seen to lie paralysed in the midline.

The inferior or recurrent laryngeal branch of the vagus nerve (X) supplies motor fibres to all the
muscles of the larynx apart from the cricothyroid muscle, and sensory fibres to the larynx inferior
to the vocal cords. Injury to this nerve will result in paralysis of all muscles of the larynx except
cricothyroid, and paralysis of the vocal cord.

-------------------------------------------------------------------------------------------------------------------------------------------------------------------------------------------------
-------
Dr Mohammed Shamsul Islam Khan, Medical Officer, Clinical Neuro-Surgery, National Institute of Neuro-Sciences and Hospital
Sher-E-Bangla Nagar, Dhaka-1207, Bangladesh. Mobile: +880 1713 455 662, +880 1685 811979. E-mail: drsikhan@gmail.com
MyPasTest Online: MRCS A - Jan Exam 2015
01. Anatomy-Head, Neck & Spine (163Qs) 61

Paralysed vocal cords lie in the paralytic position, slightly abducted from the midline, and do not
move with phonation. The sensory loss from this nerve injury will be inferior to the vocal cords on
the affected side. Cricothyroid is supplied by the superior laryngeal branch of the vagus nerve.

53. A 23-year-old male is brought by friends into A and E after falling downstairs on leaving a
nightclub. On examination, he is slightly drowsy, smelling of alcohol and complaining of neck pain.
What should be the immediate management of this patient? Page |
Analgesia YOUR ANSWER 22
C spine immobilisation CORRECT ANSWER
Immediate head and neck CT
IV access and fluid resuscitation
IV Pabrinex.

The fact a patient may be mobilising on admission to A and E should not distract from the nature of
the injury and appropriate initial management. Given the mechanism of the injury and the
complaint of neck pain, a traumatic injury to the C-spine is feasible and therefore this case should
be dealt with using ATLS principles.

54. Atherosclerosis at the carotid bifurcation is a common and potentially treatable cause of stroke.
The right common carotid artery bifurcates at what level?
Angle of mandible YOUR ANSWER
Cricoid cartilage
Jugular notch
Upper border of thyroid cartilage CORRECT ANSWER
Within the substance of the Parotid gland.

The right common carotid artery branches off the brachiocephalic artery. It bifurcates at the level of
the upper border of the lamina of the thyroid cartilage. It lies posterior to the lobes of the thyroid
gland and anterior to both the cervical sympathetic chain and the phrenic nerve on the scalenus
anterior muscle; the latter is separated from the artery by prevertebral fascia.

55. A 56-year-old heavy smoker is referred urgently to a maxillofacial surgery department with a
suspected oral cavity malignancy by his GP. Which of the following is the most common site for an
oral cavity malignancy? Select one answer only.
Hard palate YOUR ANSWER
Mucosa of vestibule
Mucosa overlying palatoglossal arch
Soft palate
Tongue CORRECT ANSWER.

Oral cavity malignancies are usually squamous cell carcinomas and 35% occur on the tongue.
They metastasise to cervical lymph nodes commonly and are related to risk factors such as
-------------------------------------------------------------------------------------------------------------------------------------------------------------------------------------------------
-------
Dr Mohammed Shamsul Islam Khan, Medical Officer, Clinical Neuro-Surgery, National Institute of Neuro-Sciences and Hospital
Sher-E-Bangla Nagar, Dhaka-1207, Bangladesh. Mobile: +880 1713 455 662, +880 1685 811979. E-mail: drsikhan@gmail.com
MyPasTest Online: MRCS A - Jan Exam 2015
01. Anatomy-Head, Neck & Spine (163Qs) 61

smoking, alcohol, malnutrition, betel nut chewing and dental caries. Surgical excision with
radiotherapy is the usual treatment of choice.

56. You are examining a patient in clinic who is referred by the GP with a swelling in the neck. The
posterior triangle of the neck contains the:
Facial nerve YOUR ANSWER Page |
Great auricular nerve 23
Omohyoid muscle CORRECT ANSWER
Roots of the brachial plexus
Vertebral artery.

The great auricular nerve turns upwards round the lateral border of the sternocleidomastoid
muscle outside the triangle. The posterior belly of the omohyoid muscle and its intermediate
tendon pass diagonally through the triangle (they are palpable, often confused with both the upper
trunk of the brachial plexus and the suprascapular nerve), and the clavicular (intermediate) and
acromial (lateral) branches of the supraclavicular nerves course through the roof.

Lying medially, outside the triangle, are the roots of the brachial plexus, sandwiched between the
scalenus anterior and medius muscles. The vertebral artery lies within the pyramidal space
inferiorly, before ascending through the foramen transversarium of the C1C6 vertebrae. Other
components of the posterior triangle include the semispinalis capitis, splenius capitis, levator
scapulae and scalenus medius muscles and the spinal accessory nerve.

57. An 82-year-old man undergoes a left carotid endarterectomy under general anaesthetic. The
operation is noted to be very difficult due to the high bifurcation of the common carotid artery.
Post-operatively has speech is difficult to understand, and it is noted on protruding his tongue, it
deviates to the left. Which nerve is most likely to be affected?
Ansacervicalis YOUR ANSWER
Facial
Glossopharyngeal
Hypoglossal CORRECT ANSWER
Spinal accessory.

The carotid sheath contains neurovascular structures and is situated between the vertebral and
visceral compartments of the neck. The sheath contains the common carotid artery, which
branches into the internal and external carotid arteries at the upper border of the thyroid cartilage.
The internal carotid artery has no branches in the neck, whereas the external carotid artery has six
branches in the neck.

The carotid body is a chemoreceptor located at the bifurcation of the common carotid artery that
monitors arterial levels of oxygen and carbon dioxide. The vagus nerve courses through the length
of the neck in the carotid sheath. The glossopharyngeal, accessory and hypoglossal nerves pass
through the superior part of the sheath in their course to the pharynx, posterior triangle and tongue
respectively.

-------------------------------------------------------------------------------------------------------------------------------------------------------------------------------------------------
-------
Dr Mohammed Shamsul Islam Khan, Medical Officer, Clinical Neuro-Surgery, National Institute of Neuro-Sciences and Hospital
Sher-E-Bangla Nagar, Dhaka-1207, Bangladesh. Mobile: +880 1713 455 662, +880 1685 811979. E-mail: drsikhan@gmail.com
MyPasTest Online: MRCS A - Jan Exam 2015
01. Anatomy-Head, Neck & Spine (163Qs) 61

The hypoglossal nerve supplies all of the muscles of the tongue except palatoglossus, which is
supplied by the pharyngeal plexus and accessory nerve. Due to its insertion on the posterolateral
aspect of the tongue, if unopposed palatoglossus will deviate the tongue laterally.

58. You see a young boy in clinic who has cleft lip and palate. Which of the following is true
regarding the incidence and type?
Page |
Isolated cleft lip is more common than combined cleft lip and palate deformity YOUR ANSWER
Left cleft lip is more common than bilateral cleft lip CORRECT ANSWER 24
Right cleft lip is more common than left
The incidence of cleft lip and palate is higher in the African population than in Caucasians
The relative risk of isolated cleft lip or combined cleft lip and palate in a child with one affected sibling
is 10%.

In Caucasians, cleft lip or cleft palate occurs in approximately 1 in 1000 live births. The incidence is
higher in Asia (1 in 500 live births) and lower in Africa (0.4 in 1000 live births). Cleft lip is thought to
be caused by either:failure of fusion between the medial nasal process and the maxillary process;
or failure of mesenchymal penetration into the layer between the ectoderm and endoderm.

Combined cleft lip and cleft palate is the most common deformity (45%). Isolated cleft lip occurs in
20% of cases and isolated cleft palate occurs in approximately 30% of cases. Left cleft lip is more
common than the right; the incidences of these are higher than bilateral cleft lip (the ratio of left:
right: bilateral cleft lip is 6:3:1).

The relative risks of a child having isolated cleft lip, or combined cleft lip and palate are:
1. no history of isolated cleft lip, or combined cleft lip and palate in the family: 0.1%
2. one affected sibling: 4%
3. two affected siblings: 9%
4. one parent and one sibling affected: 17%.

59. An anaesthetic ST4 was called to the resuscitation bay of the A&E department to help secure
the airway of a child with breathing difficulties. After recognising that the child would need to be
intubated shortly he called a senior colleague for support. Which of the following reasons make
intubating children more difficult?
Although there are no anatomical differences between adults and children the fact that children are
smaller makes intubation more difficult YOUR ANSWER
Anatomical differences between adults and children such as larger head size, a U-shaped epiglottis
and a relatively larger tongue make intubation more difficult CORRECT ANSWER
Anatomical differences between adults and children such as a more caudally placed larynx and a
smaller angle of the jaw make intubation more difficult
Intubating a child is no more difficult than intubating an adult, this trainee called a senior colleague
because he was relatively junior and needed support
Situations involving children are usually more tense because family members are often present.

Children have a relatively larger head, which tends to flex the head on the neck, making airway
obstruction more likely. The relatively larger tongue tends to flop back and obstruct the airway in
-------------------------------------------------------------------------------------------------------------------------------------------------------------------------------------------------
-------
Dr Mohammed Shamsul Islam Khan, Medical Officer, Clinical Neuro-Surgery, National Institute of Neuro-Sciences and Hospital
Sher-E-Bangla Nagar, Dhaka-1207, Bangladesh. Mobile: +880 1713 455 662, +880 1685 811979. E-mail: drsikhan@gmail.com
MyPasTest Online: MRCS A - Jan Exam 2015
01. Anatomy-Head, Neck & Spine (163Qs) 61

the obtunded child, which means that there is less room in the mouth when intubation is being
carried out.

The larynx is positioned more cephalic (glottis at C3 in infants compared with C6 in adults) and the
angle of the jaw is larger in children (140 in infants, 120 in adults), both of which make intubation
more difficult. In addition, the trachea is shorter and the cricoid ring is the narrowest part of the Page |
airway (compared with the glottis in the adult).
25
60. A 20-year-old schizophrenic man presents to a major trauma centre having been stabbed in the
neck with a screwdriver. On assessment he has no catastrophic haemorrhage, nor difficulty
breathing and a normal neurological examination. Before further assessment can be performed he
discharged himself from hospital against the medical teams advice. He presents to his GP four
weeks later complaining of a loud buzzing on the same side of his head that he was stabbed on.
What is the most likely cause of his symptoms?
This is likely to be a congenital arterio-venous fistula YOUR ANSWER
This is likely to be a post traumatic arterio-venous fistula, which requires treatment surgery to excise
the fistula and repair the artery and vein CORRECT ANSWER
This is likely to be a post traumatic arterio-venous fistula which will resolve without intervention and
follow up should be arranged to review this in 3 months
This is likely to be an auditory hallucination related to his schizophrenia. He should be referred to the
psychiatric services for review of his anti-psychotic medication
This is likely to be caused by a tracheo-oesophageal fistula, which requires surgical repair.

Arteriovenous fistulae in the neck commonly occur after penetrating trauma. The patient complains
of loud buzzing or pulsating noise in the head on the side of the fistula. Dizziness may be a feature
but neurological symptoms are usually absent. Carotidjugular fistulae are easily amenable
treatment by excision of the fistula and repair of the artery and vein. Vertebral fistulae are more
difficult to access surgically and repair is not necessary as the vertebral artery may often be
sacrificed with no danger of neurological sequelae.

Tracheo-oesophagael fistula (TOF) are usually congenital as a result of failure of fusion of the
tracheoesophageal ridges during the third week of embryological development or the result of a
surgical procedure such as a laryngectomy.

61. The sphenopalatine artery: Single best answer question choose ONE true option only.
Accompanies the infraorbital nerve to its termination YOUR ANSWER
Enters the infratemporal fossa through the pterygomaxillary fissure
Supplies blood to the lateral nasal wall and nasal septum CORRECT ANSWER
Is a terminal branch of the middle superior alveolar artery
Sends a branch into the pharyngeal canal.

-------------------------------------------------------------------------------------------------------------------------------------------------------------------------------------------------
-------
Dr Mohammed Shamsul Islam Khan, Medical Officer, Clinical Neuro-Surgery, National Institute of Neuro-Sciences and Hospital
Sher-E-Bangla Nagar, Dhaka-1207, Bangladesh. Mobile: +880 1713 455 662, +880 1685 811979. E-mail: drsikhan@gmail.com
MyPasTest Online: MRCS A - Jan Exam 2015
01. Anatomy-Head, Neck & Spine (163Qs) 61

The sphenopalatine artery, a branch of the third part of the internal maxillary artery, passes
through the sphenopalatine foramen into the cavity of the nose, at the back part of the superior
meatus. Here it gives off its posterior lateral nasal branches, which spread forward over the
conchae and meatuses, anastomose with the ethmoidal arteries and the nasal branches of the
descending palatine and assist in supplying the frontal, maxillary, ethmoidal and sphenoidal
sinuses.
Page |
Crossing the undersurface of the sphenoid, the sphenopalatine artery ends on the nasal septum as
the posterior septal branches; these anastomose with the ethmoidal arteries and the septal branch
26
of the superior labial. One branch descends in a groove on the vomer to the incisive canal and
anastomoses with the descending palatine artery.

62. A patient who had surgery in the left carotid triangle complained to his physician that he has
little sense of touch to the skin over the left side of his neck and difficulty swallowing. The patients
hyoid bone is deviated to the right side. The patients tongue is not affected. The physician
suspects that the cervical plexus of nerves to the left side of this patients neck was harmed during
the surgical procedure. Of the following nerves, which is embedded in the carotid sheath and
therefore vulnerable to injury during surgical procedures to the carotid artery? Single best answer
question choose ONE true option only.
Spinal accessory nerve YOUR ANSWER
Ansa cervicalis CORRECT ANSWER
Cervical sympathetic chain
Phrenic nerve
Suprascapular nerve.

The ansa cervicalis (or ansa hypoglossi in older literature) is a loop of nerves that are part of the
cervical plexus. Branches from the ansa cervicalis innervate the sternohyoid, sternothyroid and the
inferior belly of the omohyoid. Two roots make up the ansa cervicalis. The superior root of the ansa
cervicalis is formed by a branch of spinal nerve C1.

These nerve fibres travel in the hypoglossal nerve before leaving to form the superior root. The
superior root goes around the occipital artery and then descends embedded in the carotid sheath.
It sends a branch off to the superior belly of the omohyoid muscle and is then joined by the inferior
root. The inferior root is formed by fibres from spinal nerves C2 and C3.

63. Each branchial (pharyngeal) arch has a cartilaginous bar, a muscle component that
differentiates from the cartilaginous tissue, an artery and a cranial nerve. The first pharyngeal
(branchial) arch: Single best answer question choose ONE true option only.
Gives rise to the styloid process and hyoid bone YOUR ANSWER
Gives rise to the palatine tonsil
Gives rise to the muscles of facial expression
Gives rise to the sphenomandibular ligament CORRECT ANSWER
Is innervated by the glossopharyngeal nerve.

The first pharyngeal arch or mandibular arch is involved with development of the face. It develops
two processes, maxillary and mandibular, which form the upper and lower jaws respectively.
-------------------------------------------------------------------------------------------------------------------------------------------------------------------------------------------------
-------
Dr Mohammed Shamsul Islam Khan, Medical Officer, Clinical Neuro-Surgery, National Institute of Neuro-Sciences and Hospital
Sher-E-Bangla Nagar, Dhaka-1207, Bangladesh. Mobile: +880 1713 455 662, +880 1685 811979. E-mail: drsikhan@gmail.com
MyPasTest Online: MRCS A - Jan Exam 2015
01. Anatomy-Head, Neck & Spine (163Qs) 61

Bones and muscles of this region are developed from mesoderm in the arch. Meckels cartilage is
the first arch cartilage.

It ossifies to form the malleus and incus in the middle ear. The sphenomandibular ligament is
derived from its perichondrium. The rest of the cartilage disappears after the mandible forms
around it by intramembranous ossification. The muscles derived from the first arch include
temporalis, masseter, medial and lateral pterygoids, anterior belly of the digastric, mylohyoid, Page |
tensor tympani and tensor palati. The trigeminal nerve is the motor supply of the mandibular arch.
27
64. Which of the following statements is CORRECT regarding the muscles of facial expression?
Single best answer question choose ONE true option only.
The digastric muscle is a muscle of facial expression YOUR ANSWER
Some muscles of facial expression are innervated by the long buccal nerve
They are in the same subcutaneous plane as the platysma muscle CORRECT ANSWER
Some muscles of facial expression receive their motor supply via the zygomaticofacial nerve
All muscles of facial expression are attached to bone.

The facial muscles are subcutaneous (just under the skin, in the same plane as the platysma)
muscles that control facial expression. They generally originate on bone and insert on the skin of
the face. The facial muscles are innervated by cranial nerve VII, also known as the facial nerve. The
facial muscles are derived from the second pharyngeal arch.

65. Whilst performing an elective routine tracheostomy, heavy bleeding occurs intra-operatively.
Which of the following vessels is most likely to be the source of this?
Anterior jugular vein YOUR ANSWER
External jugular vein
Internal jugular vein
Middle thyroid vein
Superior thyroid vein.

Internal and external jugular veins, as well as the middle and superior thyroid veins, are too lateral
to be encountered in a routine tracheostomy. Anterior jugular veins and inferior thyroid veins,
however, may be encountered.

66. A 60-year-old man arrives in A&E resus with a suspected odontoid peg fracture. The odontoid
peg is:
Connected to the axis by the alar ligament YOUR ANSWER
Part of the atlas (C1 vertebrae)
Seen on open-mouth view X-ray CORRECT ANSWER
Superior to the notochord remnant
Weight bearing.

-------------------------------------------------------------------------------------------------------------------------------------------------------------------------------------------------
-------
Dr Mohammed Shamsul Islam Khan, Medical Officer, Clinical Neuro-Surgery, National Institute of Neuro-Sciences and Hospital
Sher-E-Bangla Nagar, Dhaka-1207, Bangladesh. Mobile: +880 1713 455 662, +880 1685 811979. E-mail: drsikhan@gmail.com
MyPasTest Online: MRCS A - Jan Exam 2015
01. Anatomy-Head, Neck & Spine (163Qs) 61

The axis (C2 vertebra) is characterised by the odontoid peg or dens that articulates with the atlas. It
bears no weight. The alar ligaments together with the apical ligament are attached from the sloping
upper edge of the odontoid peg to the margins of the foramen magnum.

The alar ligaments limit rotation of the head and are very strong. The apical ligament is the fibrous
remnant of the notochord and this lies superior to the odontoid peg.
Page |
67. A motorcyclist who had been involved in a high-speed accident was brought into a major
trauma centre intubated and ventilated with full spinal protection. He was unconscious. After
28
examination, he was noted to have priaprism and did not respond to painful stimuli below the
clavicle but did respond above the clavicle. The team leader suspected a cervical cord injury. What
is the most likely reason for this?
Given the mechanism of injury any assessor must be wary of a spinal cord injury YOUR ANSWER
(Correct)
Pre-hospital care teams can cause cervical cord injuries when removing the helmets of injured
motorcyclists
The patient had a blood pressure of 80/50 and a heart rate of 100 BPM
The patient had increased upper body tone
The patient had priapism and grimaced when given a painful stimulus above the clavicle but not below
it CORRECT ANSWER.

Cervical cord injury is characterised by flaccid areflexia, diaphragmatic breathing and the ability to
flex but not extend the elbow. Priapism is an uncommon but characteristic sign and is said to
evolve from the abrupt loss of sympathetic input to the pelvic vasculature leads to increased
parasympathetic input and uncontrolled arterial inflow directly into the penile sinusoidal spaces.

Full immobilisation of the neck is required at all times and pre-hospital care teams are expert at
removing helmets causing minimal movement of the cervical spine. Many motorcycle helmets are
fitted with foam inserts, which can be pulled out to assist in their removal.

Because of the loss of sympathetic tone cervical cord injuries often develop neurogenic shock with
hypotension as a result of decreased systemic vascular resistance and bradycardia because of
unopposed vagal activity. Hypotension without bradycardia in the context of trauma must be
assumed to be caused by haemorrhage until proven otherwise.

The mechanism of injury is associated with spinal column damage and injuries must be ruled out
clinically or radiologically.

In this case, the combination of priapism with loss of painful stimulus below the clavicle (i.e.
dermatome level C5 and below) is more suggestive of cervical cord injury than the other options
given.

68. A 57-year-old manual labourer attends with a rough, scaly patch on his lower lip. On taking a
history he reveals it bleeds when bumped and on examination a scaly, red patch with irregular
borders is seen. What is the commonest carcinoma of the head and neck? Select one answer only.
Basal cell carcinoma YOUR ANSWER

-------------------------------------------------------------------------------------------------------------------------------------------------------------------------------------------------
-------
Dr Mohammed Shamsul Islam Khan, Medical Officer, Clinical Neuro-Surgery, National Institute of Neuro-Sciences and Hospital
Sher-E-Bangla Nagar, Dhaka-1207, Bangladesh. Mobile: +880 1713 455 662, +880 1685 811979. E-mail: drsikhan@gmail.com
MyPasTest Online: MRCS A - Jan Exam 2015
01. Anatomy-Head, Neck & Spine (163Qs) 61

Keratoacanthoma
Melanoma
Merkel cell carcinoma
Squamous cell carcinoma CORRECT ANSWER.
Page |
Squamous-cell carcinoma (SCC) is the commonest carcinoma of the head and neck. Of the oral
cavity, the lower lip is frequently implicated, probably due to prolonged exposure to sunlight. 29
Leucoplakia is a premalignant condition and is usually painless, hence their relatively late
presentation. Lymph-node involvement is common, reflecting the usual delay in presentation and
indicates a poorer prognosis. Seventy-five per cent of lingual cancers present on the anterior two-
thirds.

69. Which of the following statements regarding the spinal arachnoid mater is CORRECT? Single
best answer question choose ONE true option only.
It is very thick compared with the cranial part YOUR ANSWER
It is separated from the dura by the subarachnoid space
It forms the denticulate ligaments
It invests the cauda equina CORRECT ANSWER
It has a rich plexus of nerves derived from the vagus.

The spinal part of the arachnoid is a thin, delicate, tubular membrane loosely investing the spinal
cord. Above, it is continuous with the cranial arachnoid. Below, it widens out and invests the cauda
equina and the nerves proceeding from it.

It is separated from the dura mater by the subdural space, but here and there this space is
traversed by isolated connective-tissue trabeculae, which are most numerous on the posterior
surface of the spinal cord.

The arachnoid surrounds the cranial and spinal nerves and encloses them in loose sheaths as far
as their points of exit from the skull and vertebral canal.

The arachnoid consists of bundles of white fibrous and elastic tissue intimately blended together.
Its outer surface is covered with a layer of low cuboidal mesothelium. The inner surface and the
trabeculae are likewise covered by a somewhat low type of cuboidal mesothelium, which in places
is flattened to a pavement type.

Vessels of considerable size, but few in number, and a rich plexus of nerves derived from the
motor root of the trigeminal, the facial and the accessory nerves, are found in the arachnoid.

70. You are in neurosurgical outpatients clinic reviewing a patient with suspected trigeminal
neuralgia. The trigeminal nerve:
Has a mandibular division which is purely sensory YOUR ANSWER
Has ophthalmic and maxillary divisions, which are purely motor
Supplies sensation to the angle of the mandible
-------------------------------------------------------------------------------------------------------------------------------------------------------------------------------------------------
-------
Dr Mohammed Shamsul Islam Khan, Medical Officer, Clinical Neuro-Surgery, National Institute of Neuro-Sciences and Hospital
Sher-E-Bangla Nagar, Dhaka-1207, Bangladesh. Mobile: +880 1713 455 662, +880 1685 811979. E-mail: drsikhan@gmail.com
MyPasTest Online: MRCS A - Jan Exam 2015
01. Anatomy-Head, Neck & Spine (163Qs) 61

Supplies the buccinator muscle


Supplies the masseter muscle CORRECT ANSWER.

The trigeminal (V) nerve has sensory fibres to the greater part of the skin of the face, mucous
membranes of the mouth, nose and paranasal air sinuses. It provides motor innervation (via its
motor, mandibular branch) to the muscles of mastication (temporalis, masseter, pterygoid). The Page |
buccinator muscle is supplied by the facial nerve. The angle of the mandible is supplied by the
great auricular nerve (C2C3). 30
71. You are performing the approach for a thyroidectomy. Which of the following is correct
regarding this approach?
The ansa cervicalis is seen readily YOUR ANSWER
The anterior jugular veins should be divided CORRECT ANSWER
The flaps raised should be superficial to platysma
The incision is made vertically at the level of the cricoid
The strap muscles are commonly divided.

The incision in thyroidectomy is made following the skin creases at about 4cm above the
sternoclavicular joints. This includes skin, subcutaneous tissue and the platysma muscle. The
anterior jugular veins, lying in the plane between the platysma are divided.

The majority of surgeons rarely divide strap muscles (namely the sternothyroid muscle at its
insertion on the thyroid cartilage) in order to gain exposure to the superior pole during
thyroidectomy.The ansa cervicalis embedded in the carotid sheath is not usually sought, unlike the
recurrent laryngeal nerves.

72. A 42-year-old male presents with chronic swelling of his jaw. An OPG shows a jaw cyst in the
body of his left mandible. What would be the preferred treatment option?
Enucleation YOUR ANSWER
Marginal mandibulatory
Marsupialisation
Segmental mandibulectomy
Wide local excision.

Jaw cysts are the most common cause of chronic swelling of the jaw. They grow slowly, displacing
rather than absorbing teeth. They are usually symptom-less unless infected, and are rarely large
enough to cause pathological fractures. Pseudocysts do not contain epithelial linings, but are still
classified as cysts.

The cysts are benign and therefore wide local excision or removal of significant amounts of
surrounding bone are not required. Both enucleation and marsupialisation are valid treatment
options and can be combined, with marsupialisation being more commonly used if infection is
present to allow this to drain, whereas enucleation removes the cyst itself.

-------------------------------------------------------------------------------------------------------------------------------------------------------------------------------------------------
-------
Dr Mohammed Shamsul Islam Khan, Medical Officer, Clinical Neuro-Surgery, National Institute of Neuro-Sciences and Hospital
Sher-E-Bangla Nagar, Dhaka-1207, Bangladesh. Mobile: +880 1713 455 662, +880 1685 811979. E-mail: drsikhan@gmail.com
MyPasTest Online: MRCS A - Jan Exam 2015
01. Anatomy-Head, Neck & Spine (163Qs) 61

73. A 70-year-old lady presents with bilateral buttock pain which worsens with prolonged standing
and improves with sitting, and also neurological claudication. Which structure of the lumbar spinal
canal is responsible for this spinal stenosis?
Normal facet joints YOUR ANSWER
Normal intervertebral disc anteriorly
Page |
Synovial facet cysts posteriorly
31
The ligamentum flavum posteriorly CORRECT ANSWER
The posterior longitudinal ligament posteriorly.

The spinal canal is anterior to the ligamentum flavum but posterior to the vertebral disc. The spinal
cord ends at L1.

74. A 78-year-old woman on home O2 for COPD has sustained a neck of femur fracture. The
anaesthetist has decided a spinal anaesthetic will be safer. What vertebral level does the spinal
cord end at in adults?
L1 - L2 YOUR ANSWER (Correct)
L3 - L4
S1 - S2
T7 - T8
T11 - T12.

The Angle of Louis is at the level of T4. The spinal dura mater ends at S2, it is the spinal cord that
ends between L1 and L2. The oesophagus traverses the diaphragm at the level of T10, the aorta
traverses the diaphragm at the level of T12.

75. A 74-year-old man is noted to have a hoarse voice following a right hemithyroidectomy. Which
nerve is most likely to have been injured?
Ansacervicalis YOUR ANSWER
Glossopharayngeal nerve
Hypoglossal nerve
Recurrent laryngeal nerve CORRECT ANSWER
Superior laryngeal nerve.

Injury to the right recurrent laryngeal nerve is a major complication of a right hemithyroidectomy.
The recurrent laryngeal nerves both ascend and lie posterior to the pretracheal fascia next to the
medial surface of the thyroid lobes. The upper root of the ansa cervicalis runs anterior to the
internal and common carotid arteries and joins the lower root. The glossopharyngeal nerve lies
between the internal jugular vein and internal carotid artery. The hypoglossal nerve lies lateral to
the occipital, internal carotid, external carotid and lingual arteries and passes over the hyoid. The
superior laryngeal nerve lies posterior, then medial to the internal carotid, then pierces the carotid
sheath to run on the wall of the pharynx.

-------------------------------------------------------------------------------------------------------------------------------------------------------------------------------------------------
-------
Dr Mohammed Shamsul Islam Khan, Medical Officer, Clinical Neuro-Surgery, National Institute of Neuro-Sciences and Hospital
Sher-E-Bangla Nagar, Dhaka-1207, Bangladesh. Mobile: +880 1713 455 662, +880 1685 811979. E-mail: drsikhan@gmail.com
MyPasTest Online: MRCS A - Jan Exam 2015
01. Anatomy-Head, Neck & Spine (163Qs) 61

76. With regards to the lumbar spine anatomy. Which part of the vertebra is responsible for causing
spondylolysis?
Intervertebral disc YOUR ANSWER
Lamina
Pars interarticularis CORRECT ANSWER Page |
Pedicle 32
Transverse process.

Spondylolysis (i.e. broken vertebra) commonly refers to a defect at the pars interarticularis of the
5th (and less frequently, the 4th) lumbar vertebra, of uncertain aetiology. On average, 5% of the
population have a spondylolysis by the age of 5 years (this rises to 6% in adults). Patients are
usually asymptomatic in the absence of slippage.

Excessive stretching may cause a spondylolisthesis, resulting in deep lumbar pain. This is
especially common in young and active adults (e.g. sportsmen who hyperextend their spine).
Neurological signs are typically absent unless localised nerve root compression is present.

77. How could you best describe the recurrent laryngeal nerve?
The recurrent laryngeal nerve supplies all of the intrinsic laryngeal muscles YOUR ANSWER
The recurrent laryngeal nerve supplies the cricothyroid muscles
The recurrent laryngeal nerves are sensory to the subglottic region and supply all of the intrinsic
muscles except the cricothyroid muscle CORRECT ANSWER
The recurrent laryngeal nerves are sensory to the supraglottic region and supplies the sternohyoid
muscle
The recurrent laryngeal nerves are sensory to the supraglottic region and supply all of the intrinsic
muscles except the cricothyroid muscle.

The recurrent laryngeal nerves are sensory to the subglottic region and supply all of the intrinsic
muscles except the cricothyroid muscle.

78. An 85-year-old man with prostatic cancer is most likely to have metastatic spread of cancer
through which of the following veins? Single best answer question choose ONE true option only.
Testicular vein YOUR ANSWER
Cephalic vein
Basilic vein
External iliac vein
Internal vertebral venous plexus CORRECT ANSWER.

The veins of the internal vertebral venous plexus are clinically significant because they are
valveless and can serve as a route for metastases. Cancerous cells can travel freely in vertebral
veins and lodge somewhere else in the body. The other veins all have valves that would direct the
flow of blood and stop some of the metastatic spread.
-------------------------------------------------------------------------------------------------------------------------------------------------------------------------------------------------
-------
Dr Mohammed Shamsul Islam Khan, Medical Officer, Clinical Neuro-Surgery, National Institute of Neuro-Sciences and Hospital
Sher-E-Bangla Nagar, Dhaka-1207, Bangladesh. Mobile: +880 1713 455 662, +880 1685 811979. E-mail: drsikhan@gmail.com
MyPasTest Online: MRCS A - Jan Exam 2015
01. Anatomy-Head, Neck & Spine (163Qs) 61

79. A 34-year-old man presents with C-spine tenderness after being involved in a road traffic
accident. He reports swerving suddenly to avoid a child on the road and colliding with a parked
car. He admits to not wearing a seat belt and injuring his chin as he fell forward into the dashboard
with his neck hyperextended. Which fracture pattern is most likely to be seen on x-ray?
Hangmans fracture YOUR ANSWER (Correct) Page |
Jefferson fracture
33
Type I Odontoid fracture
Type II Odontoid fracture
Type III Odontoid fracture.

Acute fractures of the axis, (C2 vertebra), represent about 18% of all cervico-spinal injuries and
approximately 60% of axis fractures involve the odontoid process. The odontoid process is a peg-
shaped bony protuberance that projects upward and is normally positioned in contact with the
anterior arch of C1. It is held in place primarily by the transverse ligament.

Type I odontoid fractures involve the tip of the odontoid peg, type II fractures are through the base
of the dens (involving the junction of the odontoid peg with the body) and type III fractures occur at
the base of the dens and extend obliquely into the body of the axis. Odontoid fractures are initially
identified by a lateral cervico-spinal film or open-mouth odontoid views.

In many cases, however, a computed tomography (CT) scan is required to further delineate the
type and extent of the fracture. In children younger than 6 years of age, on plain radiography, the
epiphysis may be prominent and may look like a fracture at this level. Type II is the commonest
type of odontoid fractures.

They require surgical reduction and immobilisation with a Halo and body cast. If the fracture is not
healed (and so unstable) at 12 weeks, posterior fusion of C1 to C2 may be indicated. The posterior
elements of C2, ie, the pars interarticularis may be fractured, (a hangmans fracture), by an
extension type of injury.

Patients with this fracture should be maintained in external immobilisation until specialised care is
available. These fractures represent approximately 20% of all axis fractures.

80. You examine a patients cranial nerves in clinic, turning their head to the left against direct
opposing pressure from your hand. In this motion which of the following statements is correct?
Select one answer only.
Movement is limited by cervical vertebrae YOUR ANSWER
Movement takes place at the atlanto-occipital joint
Neural impulses pass via the cranial accessory nerve
The axis of rotation runs vertically through the odontoid process CORRECT ANSWER
The left sternocleidomastoid muscle is the main agonist.

The head is turned using the contralateral sternocleidomastoid muscle, stimulated by impulses
passing in the spinal accessory nerve. The cranial accessory nerve fibres join the vagus nerve
-------------------------------------------------------------------------------------------------------------------------------------------------------------------------------------------------
-------
Dr Mohammed Shamsul Islam Khan, Medical Officer, Clinical Neuro-Surgery, National Institute of Neuro-Sciences and Hospital
Sher-E-Bangla Nagar, Dhaka-1207, Bangladesh. Mobile: +880 1713 455 662, +880 1685 811979. E-mail: drsikhan@gmail.com
MyPasTest Online: MRCS A - Jan Exam 2015
01. Anatomy-Head, Neck & Spine (163Qs) 61

soon after exiting the brain stem. The movement takes place at the atlantoaxial joint about a
vertical axis through the odontoid process of the axis, and is limited by the vertical alignment of
the two attachments of the contralateral sternocleidomastoid muscle (contralateral mastoid
process and sternoclavicular joint).

81. A 69-year-old man presents with recurrent episodes of epistaxis. What is true of epistaxis? Page |
Single best answer - select one answer only.
It is a common presentation of leukaemia YOUR ANSWER
34
It most commonly originates from the lateral nasal wall
It has no proven association with hypertension CORRECT ANSWER
Where caused by an inflammatory reaction it frequently requires treatment by submucosal resection
(SMR)
When caused by foreign bodies, the bleeding can usually be stopped by nasal packing with cotton
wool.

Epistaxis most commonly arises from the anterior part of the septum Littles area. Although
epistaxis was traditionally associated with hypertension, this association has been disproved by
various recent studies. More commonly, the anxiety associated with epistaxis produces a
hypertensive picture in such patients.

Treatment should be focused on controlling haemorrhage and reducing anxiety as primary means
of blood pressure reduction. Treatment may require nasal/post-nasal packing. Nasal packing is
performed using 1-inch ribbon gauze which may be soaked in 5% cocaine and adrenaline (1/1000).
Surgical treatment is rarely necessary and SMR is used if bleeding is from behind a septal spur or
if deviation prevents packing.

82. During submandibular gland excision a branch of the facial nerve is thought to be injured.
Which of the following manifestations is likely to occur?
Corneal ulceration YOUR ANSWER
Drooping of the corner of the mouth CORRECT ANSWER
Dry mouth
Tongue ulceration
Weakness in tongue movements.

Seventy-five per cent of cases of acute facial nerve palsy are of unknown aetiology (Bells palsy);
hence computed tomography (CT) scanning is not usually required in the presence of few clinical
signs. The HouseBrackmann scale of I (normal) to IV (complete paralysis) gives an indication of
severity, with eye closure the most important observation as this can lead to corneal ulceration
(grade III and above). Marginal mandibular nerve damage in submandibular gland surgery
manifests as drooping of the corner of the mouth. Palsy following superficial parotidectomy is in
the region of 0.5%.

-------------------------------------------------------------------------------------------------------------------------------------------------------------------------------------------------
-------
Dr Mohammed Shamsul Islam Khan, Medical Officer, Clinical Neuro-Surgery, National Institute of Neuro-Sciences and Hospital
Sher-E-Bangla Nagar, Dhaka-1207, Bangladesh. Mobile: +880 1713 455 662, +880 1685 811979. E-mail: drsikhan@gmail.com
MyPasTest Online: MRCS A - Jan Exam 2015
01. Anatomy-Head, Neck & Spine (163Qs) 61

83. A 50-year-old patient with a malignant submandibular tumour is undergoing a submandibular


gland excision with which you are assisting. Which of these structures lies lateral to the superficial
part of the submandibular gland?
Cervical branch of the facial nerve YOUR ANSWER
Deep lingual vein
Page |
Facial artery CORRECT ANSWER
35
Lingual nerve
Mylohyoid.

The facial artery, medial pterygoid and submandibular fossa of the mandible lie lateral to the
superficial part of the gland. The facial vein and cervical branch of the facial nerve lie inferior.
Mylohyoid, hyoglossus, the lingual nerve, the submandibular ganglion, the hypoglossal nerve, and
the deep lingual vein lie medially.

84. During a superficial parotidectomy the facial nerve is inadvertently injured. Which one of the
following problems might the patient experience?
Dry mouth YOUR ANSWER
Loss of corneal reflex CORRECT ANSWER
Loss of taste from anterior two thirds of tongue
Loss of taste from posterior third of tongue
Numbness of cheek.

Complications of parotidectomy are haematoma formation, wound infection, greater auricular


nerve injury (numbness of the earlobe and angle of the jaw), facial nerve palsy, gustatory sweating
(Frey's syndrome) and a salivary fistula. Because of the presence of other major and minor salivary
glands, a dry mouth is not a commonly encountered problem.

The efferent arm of the corneal reflex involves the facial nerve and so the corneal reflex may be
affected (note that the afferent arm of the corneal reflex is through the trigeminal nerve). It is
important to note that taste fibres to the anterior two thirds of the tongue extend from the chorda
tympani via the lingual nerve and are less likely to be damaged during superficial parotidectomy.

85. What structure divides the posterior triangle of the neck further into an upper and lower
triangle? Single best answer question choose ONE true option only.
Anterior belly of the digastric YOUR ANSWER
Inferior belly of the omohyoid CORRECT ANSWER
Posterior belly of the digastric
Scalenus anterior muscle
Superior belly of the omohyoid.

-------------------------------------------------------------------------------------------------------------------------------------------------------------------------------------------------
-------
Dr Mohammed Shamsul Islam Khan, Medical Officer, Clinical Neuro-Surgery, National Institute of Neuro-Sciences and Hospital
Sher-E-Bangla Nagar, Dhaka-1207, Bangladesh. Mobile: +880 1713 455 662, +880 1685 811979. E-mail: drsikhan@gmail.com
MyPasTest Online: MRCS A - Jan Exam 2015
01. Anatomy-Head, Neck & Spine (163Qs) 61

The posterior triangle is bounded in front by the sternocleidomastoid; behind, by the anterior
margin of the trapezius; its base is formed by the middle third of the clavicle; its apex, by the
occipital bone. The space is crossed, about 2.5 cm above the clavicle, by the inferior belly of the
omohyoid, which divides it into two triangles, an upper or occipital and a lower or subclavian.

It contains the accessory nerve, which crosses the triangle from the upper third of
sternocleidomastoid to the lower two-thirds of trapezius. It is particularly vulnerable to damage Page |
during lymph node biopsy, when damage results in an inability to shrug the shoulders or raise the
arm above the head (brushing hair).
36
86. The ophthalmic artery emerges through which of the following foramina to reach the eye?
Single best answer question choose ONE true option only.
Optic canal YOUR ANSWER (Correct)
Foramen spinosum
Superior orbital fissure
Foramen rotundum
Inferior orbital fissure.

The ophthalmic artery arises from the internal carotid, just as that vessel is emerging from the
cavernous sinus, on the medial side of the anterior clinoid process and enters the orbital cavity
through the optic foramen (canal), below and lateral to the optic nerve.

It then passes over the nerve to reach the medial wall of the orbit and thence horizontally forward,
beneath the lower border of the superior oblique and divides it into two terminal branches, the
frontal and dorsal nasal.

As the artery crosses the optic nerve it is accompanied by the nasociliary nerve and is separated
from the frontal nerve by the rectus superior and levator palpebrae superioris.

87. A 21-year-old boxer presents with a swollen, deformed nose after being knocked out in a fight.
He is concerned it is broken and wants you to do something about it. What is the most appropriate
management plan?
Attempt early reductions within 48 hours YOUR ANSWER
Consider reduction after 5 days CORRECT ANSWER
CT
Frontal x-ray
Lateral x-ray.

The diagnosis of nasal fractures is a clinical one, and X-rays are not usually required. Reduction is
required if there is significant cosmetic or functional abnormality, and this is usually carried out 5
days after injury to allow for swelling to subside. Nasal septal haematoma leads to cartilage
necrosis if bilateral, and causes saddle-nose deformity.

88. A 58-year-old manual labourer presents with a lesion on his lower lip, he describes a sore
which bleeds intermittently and isnt healing. On examination he has a small ulcerated lesion on

-------------------------------------------------------------------------------------------------------------------------------------------------------------------------------------------------
-------
Dr Mohammed Shamsul Islam Khan, Medical Officer, Clinical Neuro-Surgery, National Institute of Neuro-Sciences and Hospital
Sher-E-Bangla Nagar, Dhaka-1207, Bangladesh. Mobile: +880 1713 455 662, +880 1685 811979. E-mail: drsikhan@gmail.com
MyPasTest Online: MRCS A - Jan Exam 2015
01. Anatomy-Head, Neck & Spine (163Qs) 61

his lower lip approximately 1x1cm, and he is noted to have palpable lymphadenopathy of the
jugulodigastric nodes. What is the most likely cause of this lesion?
Basal cell carcinoma YOUR ANSWER
Keratoacantoma
Leukoplakia Page |
Melanoma
37
Squamous cell carcinoma CORRECT ANSWER.

SCC is the most common malignant oral cavity cancer. Cervical nodal involvement is common at
presentation, but usually involves anterior triangle nodes first, before spreading to posterior
triangle nodes. Risk factors include smoking, betel nut chewing, alcohol abuse and leucoplakia.

89. A 23-year-old male is brought into A&E as a trauma call after being stabbed in the neck by a
rival gang member. The knife has been left in place and is entering through sternocleidomastoid on
the left below the cricoid cartilage. His heart rate is 80 and blood pressure 140/90. He is alert and
able to speak in sentences. What is your management plan?
Barium swallow YOUR ANSWER
CT angiogram of head and neck CORRECT ANSWER
Endoscopy
Immediate transfer to theatre for exploration
Remove knife in A&E and pack wound.

The patient in this question has sustained a penetrating neck injury to zone 1, and is
haemodynamically stable at present as judged by the observations given. It is not necessary to
take the patient immediately to theatre, and imaging the affected area can assess the extent of
damage and help plan the nature of intervention required. In zone 1 injuries as the thoracic inlet is
involved, the thorax may need to be surgically explored, and therefore cardiothoracic surgeons
maybe needed.

The zones of penetrating neck injury are:


Zone 1 - Clavicle to cricoid cartilage
Zone 2 - Cricoid cartilage to angle of mandible
Zone 3 - Angle of mandible to base of skull

90. You are reading a radiology report before the head and neck cancer MDT. A lesion is described
as being on the 'True vocal cords' on an MRI of the neck. The true vocal cords are:
Abducted by the lateral cricoarytenoid muscles YOUR ANSWER
Abducted by the posterior cricoarytenoid muscles CORRECT ANSWER
Formed by the lower free edge of the quadrangular membranes
Innervated by sensory fibres of the internal laryngeal nerves
Lined by respiratory epithelium.

-------------------------------------------------------------------------------------------------------------------------------------------------------------------------------------------------
-------
Dr Mohammed Shamsul Islam Khan, Medical Officer, Clinical Neuro-Surgery, National Institute of Neuro-Sciences and Hospital
Sher-E-Bangla Nagar, Dhaka-1207, Bangladesh. Mobile: +880 1713 455 662, +880 1685 811979. E-mail: drsikhan@gmail.com
MyPasTest Online: MRCS A - Jan Exam 2015
01. Anatomy-Head, Neck & Spine (163Qs) 61

The true vocal folds have a stratified squamous epithelium, innervated by the recurrent laryngeal
branch (CN X), and are formed by the vocal ligament (the free edge of the quadrangular membrane
forms the false vocal cord). Above the vocal cords, the larynx is sensorily innervated by the
internal laryngeal nerve (CN X).

The cords are adducted by the lateral cricoarytenoid muscle, abducted by the posterior
cricoarytenoid and tensed by tilting the thyroid cartilage downwards and forwards by contracting Page |
the cricothyroid muscle. All the laryngeal muscles are supplied by the recurrent laryngeal nerve
except for cricothyroid, which is supplied by the external laryngeal nerve.
38
91. A 22-year-old student is brought into A&E in severe pain around his jaw. On examination his
mouth is wide open and he cannot close it. His friend reports the pain developed whilst he was
yawning and subsequently he has been like this. He is diagnosed as having a dislocated
temporomandibular joint which is reduced in A&E. Which are the following muscles is most
important in the active opening of the mouth? Select one answer only.
Buccinator YOUR ANSWER
Lateral pterygoid CORRECT ANSWER
Masseter
Medial pterygoid
Temporalis.

The muscles of mastication medial pterygoid, lateral pterygoid, masseter and temporalis all
derive from the 1stbranchial arch and are supplied by the mandibular branch of the trigeminal
nerve. Buccinator is a muscle of the cheek and aids in chewing by pressing the cheek against the
molar teeth. It is not classed as a muscle of mastication though and is supplied by the facial nerve.

Of the muscles of mastication the lateral pterygoid muscle arises by two heads: one from the
infratemporal surface of the skull and the lower from the lateral surface of the lateral pterygoid
plate. It is supplied by the anterior division of the mandibular nerve. It is indispensable to active
opening of the mouth drawing the condyle and disc forwards from the mandibular fossa.

The second branchial arch forms the hyoid bone. The pterygoid venous plexus is a network of very
small veins around and within the lateral pterygoid muscle. It drains into a pair of large short
maxillary veins.

92. You examine a patient in A&E who has sustained facial injuries in an RTA, he has mobility of
the maxillary segment on examination, and the x-rays show a fracture line passing transversely
across the maxillary sinus and pterygoid plates. This type of fracture is:
Le Fort 1 YOUR ANSWER
Le Fort 2
Le Fort 3
Le Fort 4
Craniofacial dysfunction.

-------------------------------------------------------------------------------------------------------------------------------------------------------------------------------------------------
-------
Dr Mohammed Shamsul Islam Khan, Medical Officer, Clinical Neuro-Surgery, National Institute of Neuro-Sciences and Hospital
Sher-E-Bangla Nagar, Dhaka-1207, Bangladesh. Mobile: +880 1713 455 662, +880 1685 811979. E-mail: drsikhan@gmail.com
MyPasTest Online: MRCS A - Jan Exam 2015
01. Anatomy-Head, Neck & Spine (163Qs) 61

The fractures of the maxilla can be classified into Le Fort fracture classes 1 to 3. Mobility of the
maxillary segment is the pathognomonic sign of Le Fort fracture. Clinically, it is demonstrated by
stabilising the glabella with one hand while distracting the alveolus with the other. In Le Fort 1
fracture, the fracture line passes transversely across the base of the piriform aperture, the base of
the maxillary sinus and the pterygoid plates.

Movement of the alveolus and nasofrontal region suggests a Le Fort 2 fracture; movement of the Page |
alveolus alone suggests Le Fort 1 fracture. In Le Fort 2 fracture, the bony fragment contains the
lacrimal crests, the bulk of the maxilla, the piriform margin, the alveolus and the palate. Le Fort 3
39
fracture is associated with detachment of the entire midfacial skeleton from the cranial base. It is
known as craniofacial dysfunction

93. Postganglionic sympathetic fibres innervating the dilator pupillae muscle begin in the: Single
best answer question choose ONE true option only.
Ciliary ganglion YOUR ANSWER
Superior cervical ganglion CORRECT ANSWER
Brain
Trigeminal ganglion
Spinal cord (T1-L2).

The cervical portion of the sympathetic trunk consists of three ganglia, named according to their
positions as the superior, middle and inferior ganglia and connected by intervening cords. This
portion receives no white rami communicantes from the cervical spinal nerves. Its spinal fibres are
derived from the white rami of the upper thoracic nerves and enter the corresponding thoracic
ganglia of the sympathetic trunk, through which they ascend into the neck.

The superior cervical ganglion, the largest of the three, is located opposite the second and third
cervical vertebrae. It is reddish-grey in colour and usually fusiform in shape. It is thought to be
formed by the coalescence of four ganglia, corresponding to the upper four cervical nerves.

It is related anteriorly to the sheath of the internal carotid artery and the internal jugular vein and
posteriorly to the longus capitis muscle. It contains neurones that supply sympathetic innervation
to the face (including the dilator pupillae muscle of the iris).

94. A 21-year-old male is rushed into resus after being stabbed in the back whilst on a night out.
The injury is shown on MRI to involve a right hemisection of his spinal cord. If the spinothalamic
tract is transected in the injury, what deficit will it cause below the lesion?
Loss of fine touch and proprioception on the right side YOUR ANSWER
Loss of fine touch and propriception on the left side
Loss of movement on the right side
Loss of pain and temperature on the right side
Loss of pain and temperature on the left side CORRECT ANSWER.

The spinothalamic tract conveys pain, temperature, touch and pressure sensations from one side
of the body to the opposite side of the brain. Vibration and position sense are conveyed via the
-------------------------------------------------------------------------------------------------------------------------------------------------------------------------------------------------
-------
Dr Mohammed Shamsul Islam Khan, Medical Officer, Clinical Neuro-Surgery, National Institute of Neuro-Sciences and Hospital
Sher-E-Bangla Nagar, Dhaka-1207, Bangladesh. Mobile: +880 1713 455 662, +880 1685 811979. E-mail: drsikhan@gmail.com
MyPasTest Online: MRCS A - Jan Exam 2015
01. Anatomy-Head, Neck & Spine (163Qs) 61

posterior column. The first neurone of the spinothalamic tract synapses in the posterior horn; the
next neurone crosses to the right side of the spinal cord and synapse in the thalamus, after
ascending through the cord and brainstem; the third neurone arises in the thalamus to pass to the
cortex.

The secondary axons of the spinothalamic tract ascend through the brainstem to synapse in the Page |
thalamus. Axons from the cervical region synapse medially while axons from the lumbar region
synapse laterally. A lesion of the spinothalamic tract anywhere in the brainstem would lead to a
40
loss of pain sensations from the opposite side of the body.

Temperature and touch sensations would also be diminished from the opposite side of the body
but not totally lost because other pathways may also convey these modalities. A lesion of the
spinothalamic tract at the level of the spinal cord would lead to loss of pain sensations on the
opposite side, beginning one level below the level of the lesion.

95. A 27-year-old is brought into Accident & Emergency by ambulance following a high speed road
traffic accident. What is the best method to immobilise his cervical spine?
With at least a rigid collar in all patients involved in traffic collisions of over 30 miles per hour YOUR
ANSWER
With a soft collar in all patients with blunt trauma above the clavicles
With a rigid collar, sandbags and tape in all unconscious blunt trauma patients CORRECT ANSWER
In the position that it is found until a fracture has been excluded radiologically
Manually with in-line traction until it has been secured with a collar, sandbags and tape in all
unconscious blunt trauma patients.

The cervical spine must be immobilised in all unconscious victims of trauma, those with blunt
injury above the clavicle and those with multisystem trauma. The neck is initially immobilised
manually in the in-line position NOT with traction.

The head and neck may be carefully moved into the in-line position if found in a different position,
however if any resistance is encountered it is then immobilised in the position that it was found.
For immobilisation to be adequate a collar, sandbags and tape must all be used.

96. You are assisting your consultant in a parotidectomy for suspected cancer and he is asking
you about the anatomical structures you encounter during the operation. The parotid gland:
Has a duct that passes posterior and deep to the masseter YOUR ANSWER
Is encapsulated by an investing layer of superficial cervical fascia
Is separated from the carotid sheath by the styloid process CORRECT ANSWER
Is traversed by the facial artery
Receives its blood supply directly from the internal carotid artery.

The fibrous capsule of the parotid gland is an upward extension of the deep investing layer of
cervical fascia that attaches to the zygomatic arch. The medial wall of the capsule is separated
-------------------------------------------------------------------------------------------------------------------------------------------------------------------------------------------------
-------
Dr Mohammed Shamsul Islam Khan, Medical Officer, Clinical Neuro-Surgery, National Institute of Neuro-Sciences and Hospital
Sher-E-Bangla Nagar, Dhaka-1207, Bangladesh. Mobile: +880 1713 455 662, +880 1685 811979. E-mail: drsikhan@gmail.com
MyPasTest Online: MRCS A - Jan Exam 2015
01. Anatomy-Head, Neck & Spine (163Qs) 61

from the carotid sheath by the styloid process and associated muscles (stylopharyngeus,
stylohyoid, styloglossus). The external carotid artery passes through the gland, supplying it as it
does so. No facial vessels are related to the gland. The facial nerve enters the gland and divides
within it into its five terminal branches. The duct passes anteriorly and superficial to the masseter.

97. A 78-year-old woman presents to A&E complaining of neck pain, without neurology, following a
fall from standing. A lateral x-ray of the neck reveals a retropharyngeal haematoma. The doctor in Page |
A&E suspects an odontoid peg fracture. What is the next most appropriate investigation? 41
A CT scan of the cervical spine YOUR ANSWER (Correct)
A CT scan of the whole spine
A magnetic resonance imaging (MRI) scan of the neck
An open mouth view x-ray
No further imaging is required, a retropharyngeal haematoma is diagnostic of an odontoid peg
fracture.

The odontoid peg is the ascension of the atlas fused to the ascension of the axis. The peg has an
articular facet at its front and forms part of a joint with the anterior arch of the atlas. It is a non-
weight-bearing joint. The alar ligaments, together with the apical ligaments, are attached from the
sloping upper edge of the odontoid peg to the margins of the foramen magnum.

The inner ligaments limit rotation of the head and are very strong. The weak apical ligament lies in
front of the upper longitudinal bone of the cruciform ligament, and joins the apex of the deltoid peg
to the anterior margin of the foramen magnum. It is the fibrous remnant of the notochord.

Prior to CT scanning an open mouth view was used to look for PEG fractures. In current practise a
CT scan is the gold standard. MRI scans are used to look for ligamentous injury and should be
considered after CT scanning. Given the low energy mechanism of injury and in the absence of
other symptoms there is no requirement to image the whole spine and this would expose the
patient to unnecessary radiation.

A retropharyngeal is not specific for a PEG fracture and can occur in trauma to the neck.

98. In head anatomy, the middle meningeal artery could be ruptured in head trauma. Which of the
following is true?
It is a branch of the maxillary artery which is a branch of the internal carotid artery YOUR ANSWER
It branches off the carotid artery in the infratemporal fossa
It supplies part of the dura mater, periosteum and skull bones
When it ruptures in head injury it causes extradural haematoma CORRECT ANSWER
It passes through the foramen ovale.

The middle meningeal artery is a branch of the maxillary artery in the infratemporal fossa. The
maxillary artery enters the infratemporal fossa before running forwards between the heads of
-------------------------------------------------------------------------------------------------------------------------------------------------------------------------------------------------
-------
Dr Mohammed Shamsul Islam Khan, Medical Officer, Clinical Neuro-Surgery, National Institute of Neuro-Sciences and Hospital
Sher-E-Bangla Nagar, Dhaka-1207, Bangladesh. Mobile: +880 1713 455 662, +880 1685 811979. E-mail: drsikhan@gmail.com
MyPasTest Online: MRCS A - Jan Exam 2015
01. Anatomy-Head, Neck & Spine (163Qs) 61

lateral pterygoid passing into the pterygopalatine fossa. The middle meningeal artery passes
through the foramen spinosum. The dura mater is supplied by the anterior, middle and posterior
meningeal arteries.

99. A 29-year-old male presents to clinic concerning a lump on the posterior third of his tongue.
Which of the following is this most likely to represent? Select one answer only. Page |
Filiform papillae YOUR ANSWER
42
Fungiform papillae
Lymphoid tissue CORRECT ANSWER
Palatine tonsil
Vallate papillae.

Three types of papillae are present on the upper surface of the anterior two-thirds of the tongue,
namely: filiform, fungiform and (circum) vallate papillae. The mucous membrane covering the
posterior one-third of the tongue is devoid of papillae but has a nodular irregular surface caused
by the presence of underlying lymphatic tissue, the lingual tonsil. The palatine tonsils lie lateral to
the tongue posterior to the palatoglossal arches.

100. A 20-year-old male was hit on the side of the head by a cricket ball. He initially lost
consciousness for a few seconds but then carried on with the game. Later that afternoon he
collapsed. What is the most likely diagnosis?
An extra-dural haematoma caused by damage to the anterior branch of the middle meningeal artery
YOUR ANSWER
A sub-dural haematoma caused by damage to the bridging veins on the surface of the brain
A sub-dural haematoma caused by damage to the parenchymal surface of the brain
An extra-dural haematoma caused by damage to the posterior branch of the middle meningeal artery
Post-traumatic amnesia.

Bleeding from the middle meningeal artery following head injury usually leads to an extradural
haematoma. This is usually a tear of the anterior branch of the middle meningeal artery, with an
underlying linear skull fracture. The characteristic picture is of a head injury with a brief episode of
unconsciousness followed by a lucid interval. The patient then develops a progressive
hemiparesis, stupor and rapid transtentorial coning with an ipsilateral dilated pupil. Bilateral fixed
dilated pupils, tetraplegia and death follow this.

Subdural haematomas do not present in this manner. In a young patient the blood usually arises
from the damaged brain surface. In these cases there is underlying brain injury and so no 'lucid'
interval.

Post traumatic amnesia refers to a state of confusion that occurs immediately following a traumatic
brain injury. The patient is disorientated and unable to remember events that occur after the injury.
The duration of PTA has been shown to be a good indicator of overall outcome following TBI.
-------------------------------------------------------------------------------------------------------------------------------------------------------------------------------------------------
-------
Dr Mohammed Shamsul Islam Khan, Medical Officer, Clinical Neuro-Surgery, National Institute of Neuro-Sciences and Hospital
Sher-E-Bangla Nagar, Dhaka-1207, Bangladesh. Mobile: +880 1713 455 662, +880 1685 811979. E-mail: drsikhan@gmail.com
MyPasTest Online: MRCS A - Jan Exam 2015
01. Anatomy-Head, Neck & Spine (163Qs) 61

101. A 54-year-old woman presents with longstanding tinnitus and evidence of a unilateral fifth
(trigeminal nerve) palsy. Her MRI scan shows evidence of an acoustic neuroma. Where is
compression of the trigeminal nerve most likely to be occurring? Single best answer question
choose ONE true option only.
Cerebellopontine angle YOUR ANSWER Page |
Cavernous sinus 43
Brainstem
Skull base
Trigeminal ganglion.

A complete fifth nerve lesion causes unilateral sensory loss on the face, tongue and buccal
mucosa. When motor fibres are damaged there is deviation of the jaw to the side of the lesion as
the mouth is opened. Loss of the corneal reflex may be an early indication of a fifth nerve lesion.

Brainstem lesions involving the fifth nerve nuclei may include brainstem glioma, multiple sclerosis,
brainstem infarction or syringobulbia. Lesions at the cerebellopontine angle resulting in fifth nerve
damage may include acoustic neuroma, meningioma and secondary tumour deposits.

Within the cavernous sinus, the trigeminal ganglion may be compressed by a pituitary tumour
extending into the sinus, internal carotid artery aneurysm, cavernous sinus thrombosis or
secondary tumour. The trigeminal ganglion may also be affected by herpes zoster infection.
Prognosis for the recovery of trigeminal nerve function is dependent on the underlying cause.

102. The vocal ligaments: Single best answer question choose ONE true option only.
Are formed by the superior free edge of the conus elasticus YOUR ANSWER
Are formed by the junction of the aryepiglottic and thyroepiglottic ligaments
Are formed by the inferior free edge of the thyrohyoid ligament
Contain the cricothyroid muscle
Are attached to the superior margin of the epiglottis.

The vocal folds are concerned with the production of sound and enclose two strong bands, the
vocal ligaments. Each ligament consists of a band of yellow elastic tissue formed by the superior
free edge of the conus elasticus, attached in front to the angle of the thyroid cartilage and behind
to the vocal process of the arytenoid.

Its lower border is continuous with the thin lateral part of the conus elasticus. Its upper border
forms the lower boundary of the ventricle of the larynx. Laterally, the vocalis muscle lies parallel
with it. It is covered medially by mucous membrane, which is extremely thin and closely adherent
to its surface.

103. The foramen magnum is one of several openings at the base of the skull. Which important
neurological structure/s pass through the foramen magnum?
Fascial nerve VII YOUR ANSWER
-------------------------------------------------------------------------------------------------------------------------------------------------------------------------------------------------
-------
Dr Mohammed Shamsul Islam Khan, Medical Officer, Clinical Neuro-Surgery, National Institute of Neuro-Sciences and Hospital
Sher-E-Bangla Nagar, Dhaka-1207, Bangladesh. Mobile: +880 1713 455 662, +880 1685 811979. E-mail: drsikhan@gmail.com
MyPasTest Online: MRCS A - Jan Exam 2015
01. Anatomy-Head, Neck & Spine (163Qs) 61

Hypoglossal nerve XII


Optic nerve II
The medulla oblongata and the spinal accessory nerve CORRECT ANSWER
Vestibulocochlear nerve VIII.
Page |
The structures passing through the foramen magnum include: the medulla oblongata; meninges;
spinal parts of the accessory nerves; meningeal branches of the upper cervical nerves; the 44
vertebral arteries; and the anterior and posterior spinal arteries.

104. The nasolacrimal duct empties into the: Single best answer question choose ONE true
option only.
Sphenoethmoidal recess YOUR ANSWER
Inferior meatus CORRECT ANSWER
Middle meatus
Maxillary sinus
Infundibulum.

The nasolacrimal duct is a membranous canal, about 18 mm in length, which extends from the
lower part of the lacrimal sac to the inferior meatus of the nose, where it ends by a somewhat
expanded orifice, provided with an imperfect valve, the plica lacrimalis (Hasners fold), formed by a
fold of the mucous membrane.

It is contained in an osseous canal, formed by the maxilla, the lacrimal bone and the inferior nasal
concha. It is narrower in the middle than at either end and is directed downward, backward and a
little lateralward. The mucous lining of the lacrimal sac and nasolacrimal duct is covered with
columnar epithelium, which in places is ciliated.

105. Which one of the following is true regarding the thyroid gland? Select one answer only.
Derives its arterial supply solely from branches of the external carotid YOUR ANSWER
Has a venous plexus draining into the internal jugular and brachiocephalic veins CORRECT
ANSWER
Initially moves down on swallowing
Lies superficial to the myofascial layer in the neck
Receives 20% of the total cardiac output.

The thyroid gland lies deep to the myofascial layer (strap muscles and investing layer of deep
cervical fascia), closely applied to the thyroid and cricoid cartilages. The gland initially moves up
on swallowing before returning to its normal position. The thyroid gland is highly vascular,
normally accounting for 5% of cardiac output.

While the superior thyroid artery is a branch of the external carotid artery, the inferior thyroid artery
arises from the subclavian artery, via the thyrocervical trunk. There may also be a thyroideaima
artery that may arise from the aortic arch, the brachiocephalic artery or even the internal mammary
artery. The thyroid venous plexus usually drains via three pairs of veins: the superior and middle
-------------------------------------------------------------------------------------------------------------------------------------------------------------------------------------------------
-------
Dr Mohammed Shamsul Islam Khan, Medical Officer, Clinical Neuro-Surgery, National Institute of Neuro-Sciences and Hospital
Sher-E-Bangla Nagar, Dhaka-1207, Bangladesh. Mobile: +880 1713 455 662, +880 1685 811979. E-mail: drsikhan@gmail.com
MyPasTest Online: MRCS A - Jan Exam 2015
01. Anatomy-Head, Neck & Spine (163Qs) 61

thyroid veins drain into the internal jugular; the inferior thyroid veins drain into the brachiocephalic
vein.

106. A 57-year-old male consults his Dentist with a persistent, non-healthy ulcer at the base of his
tongue which bleeds sporadically. The Dentist is concerned this could represent a squamous cell
carcinoma and refers him for urgent investigation. Which of the following are risk factors for oral
squamous cell carcinoma? Select one answer only. Page |
Diabetes mellitus YOUR ANSWER 45
HIV CORRECT ANSWER
Hypertension
Hypothyroidism
Steroid therapy.

Smoking and high alcohol intake are synergistic risk factors for oral squamous-cell carcinoma
(SCC), increasing the risk by six-fold. Other factors include syphilis, malnutrition, cirrhosis,
PlummerVinson syndrome and leucoplakia. Eighty per cent of acquired immunodeficiency
syndrome (AIDS) sufferers develop hairy oral leucoplakia.

107. A 45-year-old man presents to clinic with pain and parotid swelling which is worse with eating.
You palpate a stone in the parotid duct. The parotid duct:
Has an opening located on the floor of the mouth next to the frenulum YOUR ANSWER
Has an opening opposite the first lower molar
Is approximately 1 cm long
Lies in the middle third of a line between the intertragic notch of the auricle and the mid-point of the
philtrum CORRECT ANSWER
Runs between mylohyoid and hyoglossus.

The parotid duct is approximately 5 cm long. It crosses the masseter, turning around its anterior
border to pass through the buccal fat pad and pierce the buccinator. When intraoral pressure is
raised the submucous part of the parotid duct is compressed by the buccinator. The parotid gland
is mainly a serous gland.

The parotid duct opens on the middle third of a line between the intertragic notch of the auricle and
the midpoint of the philtrum, opposite the second upper molar. The submandibular duct opening is
located on the floor of the mouth, next to the frenulum and this duct runs between mylohyoid and
hyoglossus.

108. A 61-year-old male who is a keen gardener presents with a lesion on his lower lip which bleeds
intermittently for several months. On examination, it is a red area with thickened skin. Further
examination reveals enlarged, non-tender lymph nodes in the anterior cervical chain. A punch
biopsy confirms a squamous cell carcinoma. What is the most likely first step in the management
of lymph node metastasis? Select one answer only.
Chemotherapy YOUR ANSWER
Chemoradiotherapy
-------------------------------------------------------------------------------------------------------------------------------------------------------------------------------------------------
-------
Dr Mohammed Shamsul Islam Khan, Medical Officer, Clinical Neuro-Surgery, National Institute of Neuro-Sciences and Hospital
Sher-E-Bangla Nagar, Dhaka-1207, Bangladesh. Mobile: +880 1713 455 662, +880 1685 811979. E-mail: drsikhan@gmail.com
MyPasTest Online: MRCS A - Jan Exam 2015
01. Anatomy-Head, Neck & Spine (163Qs) 61

Radical neck dissection CORRECT ANSWER


Radiotherapy
Wide local excision.

Squamous cell carcinoma (SCC) of the mouth is related to betel nut chewing, common in Asia. Page |
Lymph node metastasis is treated with radical neck dissection, usually followed by radiotherapy.
Similarly primary excision is usually followed by radiotherapy. 46
Leucoplakia is a risk factor, especially if associated with severe epithelial dysplasia. Due to the
genetic basis of cancer, anyone is at risk; however it is rare in non-smokers.

109. A 68-year-old male is seen in a routine outpatient follow up 6 weeks after a right
submandibular gland excision. Movement of his face, neck and tongue are all normal, but he is
noted to have ulceration on the right lateral border of the tongue. Injury to which of the following
structures could have caused this?
Hypoglossal nerve YOUR ANSWER
Inferior alveolar nerve
Lingual nerve CORRECT ANSWER
Marginal mandibular nerve
Spinal accessory nerve.

Complications of submandibular gland excision are haematoma formation, marginal mandibular


nerve palsy, lingual nerve and hypoglossal nerve damage. The lingual nerve carries sensory fibres
and taste fibres to the anterior two-thirds of the tongue. Loss of sensation would give rise to
increased tendency for tongue biting and hence ulceration.

Hypoglossal nerve palsy manifests itself in weakness of the extrinsic muscles of the tongue on the
ipsilateral side. Because of the presence of other major and minor salivary glands a dry mouth is
not a commonly encountered problem. Freys syndrome is a complication of parotidectomy. The
accessory nerve is not encountered in this procedure.

110. A 28-year-old man presents with a septic cavernous sinus thrombosis, with high fever, orbital
oedema and proptosis. The primary source of infection would most likely arise from which site?
Single best answer question choose ONE true option only.
The chin YOUR ANSWER
The occipital region
The skin over the parotid gland
The pinna of the ear
The upper lip CORRECT ANSWER.

-------------------------------------------------------------------------------------------------------------------------------------------------------------------------------------------------
-------
Dr Mohammed Shamsul Islam Khan, Medical Officer, Clinical Neuro-Surgery, National Institute of Neuro-Sciences and Hospital
Sher-E-Bangla Nagar, Dhaka-1207, Bangladesh. Mobile: +880 1713 455 662, +880 1685 811979. E-mail: drsikhan@gmail.com
MyPasTest Online: MRCS A - Jan Exam 2015
01. Anatomy-Head, Neck & Spine (163Qs) 61

The cavernous sinus lies on either side of the body of the sphenoid. Anteriorly, the ophthalmic
veins drain into the sinus and communicate with the anterior facial vein, which drains the face and
upper lip hence the danger of spread of infection from this locus.

111. The submandibular duct opens: Single best answer question choose ONE true option only.
Near the maxillary second molar YOUR ANSWER Page |
Near the mandibular first molar
47
From the incisive foramen
Near the midline in the anterior aspect of the floor of the mouth CORRECT ANSWER
Into the buccal vestibule near the mandibular ramus.

The submandibular duct (Whartons duct) is about 5 cm long and its wall is much thinner than that
of the parotid duct. It begins from numerous branches from the deep surface of the gland and runs
forward between the mylohyoid and the hyoglossus and genioglossus, then between the
sublingual gland and the genioglossus and opens by a narrow orifice on the summit of a small
papilla, at the side of the frenulum linguae, near the midline in the anterior aspect of the floor of the
mouth.

On the hyoglossus it lies between the lingual and hypoglossal nerves, but at the anterior border of
the muscle it is crossed laterally by the lingual nerve. The terminal branches of the lingual nerve
ascend on its medial side.

112. A 25-year-old motorcyclist sustains a head injury and presents with a Glasgow Coma Score of
10. He has no other complications. What should the initial investigations of this patient include?
Trauma series X-rays of chest, pelvis and cervical spine YOUR ANSWER (Correct)
Insertion of an intracranial pressure monitor
A lumbar puncture
Arterial blood gases
Electrocardiography (ECG).

Plain radiographs of the chest, pelvis and cervical spine should be performed in all patients with a
significant head injury who are unconscious and so cannot be assessed reliably.

The insertion of an intracranial pressure (ICP) monitor is indicated for diffuse axonal injuries where
patients are managed conservatively, persistent raised ICP and in the presence of systemic
complications, such as severe hypotension and hypoxia. Lumbar puncture is contraindicated in
patients with raised ICP due to the risk of coning.

113. A 71-year-old heavy smoker undergoes a Carotid endarterectomy for a recent transient
ischaemic attack (TIA). After careful dissection anterior to the sternocleidomastoid muscle the
carotid sheath is reached. Which of the following is the most anterior structure within the carotid
sheath? Select one answer only.
Ansa Cervicalis YOUR ANSWER

-------------------------------------------------------------------------------------------------------------------------------------------------------------------------------------------------
-------
Dr Mohammed Shamsul Islam Khan, Medical Officer, Clinical Neuro-Surgery, National Institute of Neuro-Sciences and Hospital
Sher-E-Bangla Nagar, Dhaka-1207, Bangladesh. Mobile: +880 1713 455 662, +880 1685 811979. E-mail: drsikhan@gmail.com
MyPasTest Online: MRCS A - Jan Exam 2015
01. Anatomy-Head, Neck & Spine (163Qs) 61

Common carotid artery


Internal carotid artery
Internal jugular vein CORRECT ANSWER
Vagus nerve.
Page |
The internal jugular vein descends the neck in the anterior aspect of the carotid sheath, and drains
into the brachiocephalic vein on each side behind the sternoclavicular joint. 48
The Vagus nerve passes vertically down within the Carotid sheath closely related to the internal
carotid artery and lying between it and the internal jugular vein. The ansa loops anteriorly around
the carotid sheath under cover of the sternocleidomastoid muscle.

114. During thyroidectomy, the inferior laryngeal branch of the right recurrent laryngeal nerve was
injured. The action of which of the following laryngeal muscles is most likely to be affected? Single
best answer question choose ONE true option only.
Thyroarytenoid YOUR ANSWER
Arytenoid
Lateral cricoarytenoid
Posterior cricoarytenoid CORRECT ANSWER
Cricothyroid.

The posterior cricoarytenoid is innervated by the inferior laryngeal nerve, which is a continuation
of the recurrent laryngeal nerve. The posterior cricoarytenoid is the only muscle that abducts the
vocal folds. If this muscle is denervated, the vocal folds may be paralysed in an adducted position,
which would prevent air from entering the trachea. Arytenoid, lateral cricoarytenoid and
thyroarytenoid all adduct the vocal folds.

Cricothyroid is the only laryngeal muscle innervated by the external branch of the superior
laryngeal. It tenses the vocal ligaments by tipping the thyroid cartilage forward relative to the
cricoid cartilage.

115. Childrens bones are more elastic than adult bone. Which particular type of fracture affects
elastic bone?
Colles fracture YOUR ANSWER
Galeazzi fracture
Monteggia fracture
Tibial tubercle apophysitis
Torus and green stick fractures CORRECT ANSWER.

The skull and long bones in young children are still ossifying and therefore more malleable than in
adults. Fractures to these areas in young children should therefore be treated with suspicion and
no accidental injury (NAI) considered. Colles fractures are classical to, although not exclusive to,
osteoporosis and therefore elderly adults.

-------------------------------------------------------------------------------------------------------------------------------------------------------------------------------------------------
-------
Dr Mohammed Shamsul Islam Khan, Medical Officer, Clinical Neuro-Surgery, National Institute of Neuro-Sciences and Hospital
Sher-E-Bangla Nagar, Dhaka-1207, Bangladesh. Mobile: +880 1713 455 662, +880 1685 811979. E-mail: drsikhan@gmail.com
MyPasTest Online: MRCS A - Jan Exam 2015
01. Anatomy-Head, Neck & Spine (163Qs) 61

116. A 25-year-old male attends hospital on a Friday night, after being involved in a fight. His nose
is clinically deformed and bleeding. Which complication of nasal fractures requires urgent
treatment?
Associated facial fracture confirmed with facial view X-rays YOUR ANSWER
Deformity of the nose Page |
Minor associated epistaxis
49
Nasal septal fracture
Nasal septal haematoma CORRECT ANSWER.

A numb cheek suggests damage to infraorbital nerve, most commonly associated with infraorbital
rim fracture, not usually nasal fracture. Bony manipulation under anaesthesia (MUA) must be
performed within the first 2 weeks of injury, otherwise reduction is difficult and the patient usually
needs a formal rhinoplasty.

Septal haematoma and subsequent abscess formation is potentially life threatening and so must
be excluded at initial examination. X-rays are not helpful in nasal fractures; epistaxis is seldom
troublesome.

117. You are reviewing a child in the ENT outpatient clinic following a tonsillectomy some weeks
ago. The palatine tonsillar bed:
Contains the superior pharyngeal nerve YOUR ANSWER
Is drained by the external palatine vein CORRECT ANSWER
Is floored by the middle constrictor muscle
Is pierced by branches of the mandibular artery
Lies in the oral cavity.

The palatine tonsil lies in the oropharynx, in a pit floored by the superior constrictor muscle,
through which the IXth cranial nerve passes. The tonsil is supplied by the tonsillar branch of the
facial artery. Venous blood first drains into the tonsillar venous plexus, then into the pharyngeal
venous plexus and thence into the external palatine vein (a bleeding point after tonsillectomy), or
the facial and pharyngeal veins.

118. Following a left carotid endarterectomy a 74-year-old patient is noted to have developed
speech problems. He seems able to understand what is said but unable to get the words out.
Which area of the brain is most likely to be affected?
Basal ganglia YOUR ANSWER
Brocas area CORRECT ANSWER
Occipital cortex
Olfactory cortex
Wernickes area.

The primary motor cortex is in the precentralgyrus and the primary sensory cortex in the
postcentral cortex. Brocas area lies in the posterior part of the inferior frontal gyrus of the
-------------------------------------------------------------------------------------------------------------------------------------------------------------------------------------------------
-------
Dr Mohammed Shamsul Islam Khan, Medical Officer, Clinical Neuro-Surgery, National Institute of Neuro-Sciences and Hospital
Sher-E-Bangla Nagar, Dhaka-1207, Bangladesh. Mobile: +880 1713 455 662, +880 1685 811979. E-mail: drsikhan@gmail.com
MyPasTest Online: MRCS A - Jan Exam 2015
01. Anatomy-Head, Neck & Spine (163Qs) 61

dominant hemisphere. Lesions to Brocas area cause expressive dysphasia, classically described
as knowing what to say, but being unable to get the words out.

The putamen and globuspallidus are together known as the lentiform nucleus and form part of the
basal ganglia. The olfactory impulses travel to the temporal lobe in the region of the uncus.

119. The facial nerve is a structure of utmost importance in head and neck surgery. Which of the Page |
following muscles is supplied by the facial nerve?
50
Anterior belly of digastric YOUR ANSWER
Buccinator CORRECT ANSWER
Medial pterygoid
Mylohyoid
Temporalis.

The buccal branch of the nerve pierces the buccinator muscle after supplying it and the muscles of
the upper lip. The trigeminal (V) nerve is predominantly associated with the muscles of
mastication. These include, masseter, temporalis and the pterygoid muscles.

There are five main divisions of the facial nerve originating from the parotid gland and the lowest
or fifth branch is the cervical. The facial nerve does pass through the stylomastoid foramen.

120. A worried mum has brought her 4-year-old son to the GP because she is concerned about his
speech which she has noticed has deteriorated. On further questioning it emerges he has some
difficulty hearing instructions and has to turn the volume up very high when watching television.
His only past medical history of note is recurrent upper respiratory tract infections. What is the
most likely cause for his symptoms? Select one answer only.
Cholesteatoma YOUR ANSWER
Glue ear CORRECT ANSWER
Mastoiditis
Otosclerosis
Presbyacusis

Many of the problems resulting from glue ear are secondary to hearing loss. In children this can
manifest in several different ways such as problems with speech and language, not hearing
instructions or turning up the volume on the television. Glue ear (otitis media with effusion) is the
most common complication of acute otitis media.

Glue ear is the commonest cause of hearing loss in young children. Enlarged adenoids, common in
children, may occlude the Eustachian tube orifice. The palatal muscles help to open the Eustachian
tube, so glue ear is common in children with a cleft palate. There is evidence confirming that
parental smoking increases the development of glue ear.

-------------------------------------------------------------------------------------------------------------------------------------------------------------------------------------------------
-------
Dr Mohammed Shamsul Islam Khan, Medical Officer, Clinical Neuro-Surgery, National Institute of Neuro-Sciences and Hospital
Sher-E-Bangla Nagar, Dhaka-1207, Bangladesh. Mobile: +880 1713 455 662, +880 1685 811979. E-mail: drsikhan@gmail.com
MyPasTest Online: MRCS A - Jan Exam 2015
01. Anatomy-Head, Neck & Spine (163Qs) 61

121. You are in the neurosurgical clinic reviewing a patient who has had a left temporal lobe
tumour resected and now has a suspected abscess. He has raised inflammatory markers, fever and
headache. A dominant hemisphere temporal lobe abscess is most likely to cause which symptom?
Motor disturbance in the contralateral arm YOUR ANSWER
Problems with balance
Problems with speech (dysphasia) CORRECT ANSWER
Page |
Sensory disturbance in the contralateral leg 51
Visual disturbance.

Cerebral abscesses act as space-occupying lesions and produce focal neurological deficits. They
may also cause meningitis, epilepsy and intracranial herniation. Dysphasia occurs with dominant
(most commonly left) temporal lobe lesions. The important cortical areas of the temporal lobe
area are the auditory cortex and temporal association area (responsible for the recognition of
auditory stimuli and integration with other modalities).

122. A 47-year-old male with a history of alcoholic liver disease presents with a six week history of
progressively worsening epigastric pain and distension. Computerised tomography (CT) reveals a
4cm fluid collection in the lesser sac. What is the likely diagnosis?
Hepatocellular carcinoma YOUR ANSWER
Liver abscess
Pancreatic pseudocyst CORRECT ANSWER
Perforation of the posterior gastric wall
Von-Hippel Lindau disease.

Pancreatic pseudocysts are a complication of both acute and chronic pancreatitis. The incidence
of all three is increased in patients with alcoholic liver disease. They usually develop within the
lesser sac, and so can be drained via a posterior wall gastrostomy at endoscopy. Von-Hippel
Lindau disease is a rare inherited pathology that causes cyst development in multiple organs,
including the pancreas.

123. A 27-year-old man is brought into A&E with facial fractures following a road traffic accident.
Which step is important in the initial management of facial injuries?
Antibiotics and tetanus immunisation are always required YOUR ANSWER
Early surgical correction is preferable
Facial X-rays are helpful and easy to interpret
Nasal intubation is always indicated
First aid and nasal cautery should be used initially to control nasal bleeding CORRECT ANSWER.

When dealing with severe facial trauma, advanced trauma life support (ATLS) principles must be
adhered to. Therefore in the initial management using the achievable benchmarks of care (ABCs)
scenarios, bleeding must be sought and stopped, especially as this can lead to airway
compromise.

-------------------------------------------------------------------------------------------------------------------------------------------------------------------------------------------------
-------
Dr Mohammed Shamsul Islam Khan, Medical Officer, Clinical Neuro-Surgery, National Institute of Neuro-Sciences and Hospital
Sher-E-Bangla Nagar, Dhaka-1207, Bangladesh. Mobile: +880 1713 455 662, +880 1685 811979. E-mail: drsikhan@gmail.com
MyPasTest Online: MRCS A - Jan Exam 2015
01. Anatomy-Head, Neck & Spine (163Qs) 61

Surgery can usually wait until the patient is fully resuscitated. Chest X-ray must be performed as
part of routine imaging, especially if the possibility of inhaled teeth is to be considered. IV
antibiotics may be necessary if there is the possibility of open fractures, but are not always
necessary.

124. You are performing a lymph node biopsy in the posterior triangle of the neck and you need to
mark out the boundaries of the posterior triangle. These are? Page |
The anterior border of sternocleidomastoid, the mandible and the midline YOUR ANSWER 52
The anterior border of sternocleidomastoid, the posterior border of trapezius and the clavicle
The anterior border of sternocleidomastoid, the posterior border of trapezius and the scapula
The posterior border of sternocleidomastoid, the anterior border of trapezius and the scapula
The posterior border of sternocleidomastoid, anterior border of trapezius and the clavicle CORRECT
ANSWER.

The posterior triangle lies between the posterior border of sternocleidomastoid, the anterior border
of trapezius and the clavicle. The anterior triangle lies between the anterior border of
sternocleidomastoid, the mandible and the midline.

Lymph node excision from the posterior triangle can result in damage to the accessory nerve. The
anterior triangle is subdivided into four further triangles. The common carotid artery bifurcates in
the anterior triangle.

125. A 72-year-old diabetic male who is a lifelong smoker is referred by his GP to a rapid access
TIA clinic because of episodes of transient vision loss affecting his left eye. In the clinic he is
thought to have had episodes of amaurosisfugax due to emboli affecting the ophthalmic artery.
The ophthalmic artery is a branch of which vessel? Select one answer only.
Anterior cerebral artery YOUR ANSWER
External carotid artery
Facial artery
Internal carotid artery CORRECT ANSWER
Middle cerebral artery.

The ophthalmic artery is a branch of the internal carotid artery. It passes through the optic canal
and supplies the ethmoidal air cells, part of the lateral wall of the nose, external nose, eyelids and
forehead. It also supplies all the muscles of the orbit.

126. A 78-year-old male taking warfarin experiences a sudden onset of headache with visual
disturbance. CT scan demonstrates a left occipital lobe intracerebral haematoma. What visual
finding would expect to find on examination? Select one answer only.
Bitemporal hemianopia YOUR ANSWER
Central scotoma
Left homonymous hemianopia
Macular sparing always

-------------------------------------------------------------------------------------------------------------------------------------------------------------------------------------------------
-------
Dr Mohammed Shamsul Islam Khan, Medical Officer, Clinical Neuro-Surgery, National Institute of Neuro-Sciences and Hospital
Sher-E-Bangla Nagar, Dhaka-1207, Bangladesh. Mobile: +880 1713 455 662, +880 1685 811979. E-mail: drsikhan@gmail.com
MyPasTest Online: MRCS A - Jan Exam 2015
01. Anatomy-Head, Neck & Spine (163Qs) 61

Right visual field loss CORRECT ANSWER.

A right homonymous hemianopia would result from a left occipital lobe infarction and the macular
would also be affected. Macular or central field sparing can occur if the occipital pole remains
intact through blood supply from a branch of the middle cerebral artery.
Page |
A central scotoma describes visual loss corresponding with the point of fixation most commonly
caused by a lesion between the optic chiasm and the retina. A bitemporal hemianopia is a sign of 53
optic chiasm compression, oftendue to pituitary gland tumours.

127. The transverse cervical artery is severed in a road traffic accident. Which muscle would be
affected the most? Single best answer question choose ONE true option only.
Levator scapulas YOUR ANSWER
Rhomboideus minor
Rhomboideus major
Trapezius CORRECT ANSWER
Latissimus dorsi.

The transverse cervical artery supplies blood to the trapezius. Levator scapulas and the rhomboids
receive blood from the dorsal scapular artery. Latissimus dorsi receives blood from the
thoracodorsal artery.

128. A 67-year-old man is brought to hospital following an assault. He has a deep laceration over
his right temporal scalp which is bleeding profusely. Which of the following describes the anatomy
of scalp most accurately?
Contains lymph nodes YOUR ANSWER
Contains the C1 dermatome
Has motor innervation supplied by the facial nerve CORRECT ANSWER
Is supplied exclusively from branches of the external carotid artery
Is tightly attached to the cranium.

From superficial to deep, the scalp has five basic layers, denoted by the mnemonic SCALP: skin;
connective tissue; aponeurosis; loose areolar tissue; and periosteum. The blood supply to the
scalp is from branches of both the internal and external carotid arteries. Two such tributaries from
the internal carotid are the supraorbital and supratrochlear arteries.

There is no C1 dermatome. The aponeurosis of the scalp is separated from the epicranium by loose
connective tissue (the plane of cleavage in scalping), facilitating gliding movements. It contains no
lymph nodes. Lymphatic drainage is mainly to the submandibular, preauricular, mastoid and
occipital nodes. The occipitofrontalis muscle is supplied by the VIIth cranial nerve.

129. In surgery of the thyroid gland, the external laryngeal nerve may be injured and must be
identified before ligating the: Single best answer question choose ONE true option only.
Inferior thyroid artery YOUR ANSWER
-------------------------------------------------------------------------------------------------------------------------------------------------------------------------------------------------
-------
Dr Mohammed Shamsul Islam Khan, Medical Officer, Clinical Neuro-Surgery, National Institute of Neuro-Sciences and Hospital
Sher-E-Bangla Nagar, Dhaka-1207, Bangladesh. Mobile: +880 1713 455 662, +880 1685 811979. E-mail: drsikhan@gmail.com
MyPasTest Online: MRCS A - Jan Exam 2015
01. Anatomy-Head, Neck & Spine (163Qs) 61

Superior thyroid artery CORRECT ANSWER


Superior pharyngeal artery
Inferior laryngeal artery
Ascending palatine artery.
Page |
The external laryngeal nerve is the smaller, external branch (ramus externus) of the superior
laryngeal nerve. It descends on the larynx, beneath the sternothyroid muscle, to supply the 54
cricothyroid muscle. It gives branches to the pharyngeal plexus and the superior portion of the
inferior pharyngeal constrictor and communicates with the superior cardiac nerve behind the
common carotid artery. The external branch is susceptible to damage during thyroidectomy, as it
lies immediately deep to the superior thyroid artery.

130. A patient with a pleomorphic adenoma of the left parotid gland requires surgical resection.
You are consenting the patient to surgery. What is the most common complication to warn him
about?
A high risk of non-compressible bleeding YOUR ANSWER
A post-operative sialocele (salivary fistula)
Loss of taste over the anterior two thirds of the tongue
Permanent facial weakness
Redness and sweating in the cheek CORRECT ANSWER.

The axons conveying taste over the anterior two-thirds of the tongue and secretomotor fibres to
the submandibular and sublingual salivary glands (the parotid is innervated by the IXth cranial
nerve) are found in the chorda tympani nerve.

The chorda tympani courses across the tympanic membrane beneath the mucous membrane of the
middle ear after leaving the facial canal approximately 0.5 cm above the stylomastoid foramen. It
exits the middle ear by passing through the petrotympanic fissure medially, and runs forwards on
the medial side of the spine of the sphenoid bone.

After entering the infratemporal fossa it merges with the lingual nerve. The chorda tympani,
therefore, is not related to the parotid gland.

The facial nerve runs in close proximity to the parotid gland and is easily damaged during surgery.
In the context of tumours the surgeon may decide to remove the nerve deliberately to reduce the
risk of recurrence. After surgery facial weakness may be present but usually resolves with time.

Freys syndrome is caused when parasympathetic innervation of the normally sympathetic


branches of the trigeminal nerve occurs. In this condition patients often develop sweating on the
side of the cheek at the anticipation of eating.

-------------------------------------------------------------------------------------------------------------------------------------------------------------------------------------------------
-------
Dr Mohammed Shamsul Islam Khan, Medical Officer, Clinical Neuro-Surgery, National Institute of Neuro-Sciences and Hospital
Sher-E-Bangla Nagar, Dhaka-1207, Bangladesh. Mobile: +880 1713 455 662, +880 1685 811979. E-mail: drsikhan@gmail.com
MyPasTest Online: MRCS A - Jan Exam 2015
01. Anatomy-Head, Neck & Spine (163Qs) 61

Whilst bleeding and post operative sialocele are recognized complications of parotid surgery their
risk can be minimized by good surgical techniques.

131. The application of arterial clips during total thyroidectomy is most likely to injure which of the
following nerves?
External laryngeal YOUR ANSWER (Correct) Page |
Hypoglossal 55
Internal laryngeal
Phrenic
Transverse cervical.

When the superior thyroid arteries are ligated, the external laryngeal nerves running alongside can
be easily damaged. When the inferior thyroid arteries are ligated, the recurrent laryngeal nerves are
also vulnerable. The internal laryngeal nerves pass above and behind the root of the superior
thyroid arteries and are therefore usually outside the operative field.

The hypoglossal nerves run in the anterior triangle of the neck superior to the hyoid bone and so
are not in proximity. The phrenic nerves are protected as they lie behind the prevertebral fascia.
Both transverse cervical nerves run in the subcutaneous fascia. Horizontal skin-crease
thyroidectomy incisions run parallel with their course, and therefore most branches of these
cutaneous nerves are spared.

132. A dental surgeon carries out a block of the inferior alveolar nerve by infiltrating local
anaesthetic at the mandibular foramen. Which clinical feature may result from this procedure?
Single best answer question choose ONE true option only.
Numbness of the lower lip on the injected side YOUR ANSWER (Correct)
Ineffective block for the incisor teeth
Numbness of hard palate
Inability of the patient to clench his jaws
Transient weakness of the facial muscles on the injected side.

The inferior alveolar nerve, a branch of the mandibular division of the trigeminal nerve (V),
traverses the inferior alveolar, or dental, canal of the mandible to supply all the teeth of that
hemimandible; all the teeth on that side are therefore anaesthetised. The mental branch of the
nerve emerges through the mental foramen to supply the lower lip, which becomes numb in a
successfully performed block.

Note that although the inferior alveolar nerve is not associated with tongue sensation,
administration of anaesthesia near the mandibular foramen partially anaesthetizes the nearby
lingual nerve by diffusion, thus resulting in ipsilateral numbness to the side of the tongue.

The muscles of the tongue, of mastication and of facial expression are not affected.

-------------------------------------------------------------------------------------------------------------------------------------------------------------------------------------------------
-------
Dr Mohammed Shamsul Islam Khan, Medical Officer, Clinical Neuro-Surgery, National Institute of Neuro-Sciences and Hospital
Sher-E-Bangla Nagar, Dhaka-1207, Bangladesh. Mobile: +880 1713 455 662, +880 1685 811979. E-mail: drsikhan@gmail.com
MyPasTest Online: MRCS A - Jan Exam 2015
01. Anatomy-Head, Neck & Spine (163Qs) 61

133. You are treating a patient who has sustained a stab wound to the face which has completely
lacerated the mandibular branch of the trigeminal nerve at its origin. Which of the following
muscles would remain unaffected?
Anterior belly of digastric YOUR ANSWER
Lateral pterygoid
Masseter
Page |
Medial pterygoid 56
Posterior belly of digastric CORRECT ANSWER.

The mandibular division of the trigeminal nerve supplies all the muscles of mastication, including
the lateral pterygoid, medial pterygoid, master and temporalis. It also supplies the anterior belly of
digastric, mylohyoid, tensor tympani and tensor veli palatini.

134. You are reviewing a patient in clinic who has been referred by their GP with a swelling in the
neck and episodes of blackouts. Your suspected diagnosis should be:
Carotid body tumour (chemodectoma) YOUR ANSWER (Correct)
Cervical rib
Laryngocele
Pharyngeal pouch
Thyroglossal cyst.

Lateral swellings of the neck include lymph nodes, salivary glands, branchial cysts, cervical ribs,
carotid body tumours(chemodectomas), cystic hygromas, sternocleidomastoid tumours,
arteriovenous fistulae and lateral thyroid lobe lesions. A chemodectoma is a tumour of the glomus
jugulare, and contains chemoreceptor tissue; it may cause attacks of vertigo and syncope.

135. Which of the following is most likely to cause a bitemporal hemianopia?


A craniopharyngioma YOUR ANSWER
A pituitary adenoma CORRECT ANSWER
A pituitary astrocytoma
A sphenoid wing meningioma
Pituitary apoplexy.

The pituitary fossa, or sella turcica, lies above the body of the sphenoid bone and its associated
sinus. The pituitary fossa can be easily identified on a lateral skull X-ray. The optic chiasm lies
above and towards the back of the sella turcica. A pituitary tumour would first impinge on the
anterior part of the optic chiasm and so cause a temporal hemianopia.

Craniopharyngiomas comprise of 1.2-3% of all brain tumours in adults. Although meningiomas can
compress the optic chiasm, one based on the sphenoid wing would cause other symptoms prior to
a hemianopia. Pituitary tumours comprise between 10-15% of all intracranial tumours.

-------------------------------------------------------------------------------------------------------------------------------------------------------------------------------------------------
-------
Dr Mohammed Shamsul Islam Khan, Medical Officer, Clinical Neuro-Surgery, National Institute of Neuro-Sciences and Hospital
Sher-E-Bangla Nagar, Dhaka-1207, Bangladesh. Mobile: +880 1713 455 662, +880 1685 811979. E-mail: drsikhan@gmail.com
MyPasTest Online: MRCS A - Jan Exam 2015
01. Anatomy-Head, Neck & Spine (163Qs) 61

Pituitary astrocytomas are incredibly rare pituitary tumours. Pituitary apoplexy is caused by
infarction of the pituitary gland caused by bleeding into a tumour, although it is a cause of
bitemporal hemianopia it is less common than visual defects because of an adenoma.

136. A 12-year-old girl is brought to her GP by her mother with a lump noticed in her neck. It is
painless and she is well in herself. On examination, it is inferior to the hyoid bone and in the
midline. On protruding her tongue, it is noted to move up. What is the most likely cause of this Page |
lump? Select one answer only.
57
Branchial cyst YOUR ANSWER (Correct)
Infectious mononucleosis
Metastatic carcinoma
Pharyngeal pouch
Thyroglossal cyst CORRECT ANSWER.

Thyroglossal cysts are found in the midline and rise up on protruding the tongue. They originate
due to the thyroid beginning in the foramen caecum at the back of the tongue in embryology and
descending to its final position.

Branchial cysts arise due to the non-disappearance of the cervical sinus (where the 2ndbranchial
arch grows down over the 3rd and 4th). They are mostly found under the anterior border of
sternocleidomastoid where the upper third meets the middle third.

Pharyngeal pouches may protrude into the posterior triangle on swallowing. These occur mostly
on the left and a pharyngeal pouch is a pulsion diverticulum that protrudes between the fibres of
the inferior pharyngeal constrictor.

137. Which of the following structures is the temporomandibular joint (TMJ) attached to superiorly?
Articular disc YOUR ANSWER
Articular eminence
Articular tubercle
Postglenoid tubercle
Squamo-tympanic fissure CORRECT ANSWER

The temporomandibular joint is a synovial joint situated between the condyle of the mandible
below and the mandibular fossa above. Although it a synovial joint, it is lined by fibrous cartilage
(rather than hyaline cartilage typical of synovial joints). The joint is surrounded by a capsule that is
attached beyond the limits of the articular surfaces.

The capsule is strengthened medially and laterally by collateral ligaments. Superiorly, the capsule
is attached to the anterior edge of the squamo-tympanic fissure. Medially, it runs along the suture
between the temporal and sphenoid bones and attaches anteriorly to the anterior end of the
articular eminence. Laterally, the capsule is attached to the articular tubercle, which forms the
lateral limit of the articular eminence and the prominent ridge of the bone forming the lateral lip of
the glenoid cavity.
-------------------------------------------------------------------------------------------------------------------------------------------------------------------------------------------------
-------
Dr Mohammed Shamsul Islam Khan, Medical Officer, Clinical Neuro-Surgery, National Institute of Neuro-Sciences and Hospital
Sher-E-Bangla Nagar, Dhaka-1207, Bangladesh. Mobile: +880 1713 455 662, +880 1685 811979. E-mail: drsikhan@gmail.com
MyPasTest Online: MRCS A - Jan Exam 2015
01. Anatomy-Head, Neck & Spine (163Qs) 61

The non-articular surfaces enclosed within the capsule are lined with synovial membrane. The joint
is stable anteriorly but is lax posteriorly as it is attached well below the articular surface to the
neck of the condyle.

138. Following parotid gland surgery in a 51-year-old female, the facial nerve is damaged. What is Page |
the most likely consequence for the patient?
58
Altered taste to the anterior 2/3 of the tongue YOUR ANSWER
Difficulty in chewing food therefore weakness in the muscle of mastication
Inability to rotate the neck because of weakness in sternocleidomastoid
Slurred speech due to weakness in tongue movements
Weakness in buccinator and orbicularis oris CORRECT ANSWER.

As part of its course, having exited the stylomastoid foramen and passed between the mastoid
process and tympanic ring, the facial nerve comes to lie between the deep and superficial portions
of the parotid gland. Therefore it is at risk of injury during parotidectomy.

Within the gland the facial nerve divides into its superior and inferior divisions. The branches given
off are the temporal, zygomatic, buccal, mandibular and cervical branches. The buccal branch
provides the nerve supply to buccinators and orbicularis oris.

Taste to the anterior two thirds of the tongue is supplied by the chorda tympani, this branch of the
facial nerve is given off prior to the nerve entering the parotid gland. The muscles of mastication
are predominantly supplied by the trigeminal nerve, (masseter, temporalis and the pterygoids).
Sternocleidomastoid is suppled by the accessory nerve. The muscles which move the tongue are:
palatoglossus, which is supplied by the pharyngeal plexus and accessory nerve, and
genioglossus, hyoglossus and styloglossus, which are supplied by the hypoglossal nerve.

139. The supratrochlear nerve is a terminal branch of which nerve? Single best answer question
choose ONE true option only.
Ophthalmic YOUR ANSWER
Frontal CORRECT ANSWER
Nasociliary
Lacrimal
Maxillary.

The supratrochlear nerve, the smaller of the two branches of the frontal nerve, passes above the
pulley of the superior oblique and gives off a descending filament to join the infratrochlear branch
of the nasociliary nerve.

It then escapes from the orbit between the pulley of the superior oblique and the supraorbital
foramen, curves up onto the forehead close to the bone, ascends beneath the corrugator and
frontalis and dividing into branches which pierce these muscles, it supplies the skin of the lower

-------------------------------------------------------------------------------------------------------------------------------------------------------------------------------------------------
-------
Dr Mohammed Shamsul Islam Khan, Medical Officer, Clinical Neuro-Surgery, National Institute of Neuro-Sciences and Hospital
Sher-E-Bangla Nagar, Dhaka-1207, Bangladesh. Mobile: +880 1713 455 662, +880 1685 811979. E-mail: drsikhan@gmail.com
MyPasTest Online: MRCS A - Jan Exam 2015
01. Anatomy-Head, Neck & Spine (163Qs) 61

part of the forehead close to the midline and sends filaments to the conjunctiva and skin of the
upper eyelid.

140. A 25-year-old male is brought to A&E after being assaulted outside a nightclub. On
examination there is considerable swelling present, and he is noted to have a reduced sensation in
the distribution of his left infraorbital nerve. A CT reveals a fracture through the nasal bones on the
ascending process of the maxilla, lacrimal bone and extending into the orbital rim. The orbital floor Page |
is otherwise intact. What is the likely injury?
59
Base of skull fracture YOUR ANSWER
Le fort I fracture
Le fort II fracture CORRECT ANSWER
Le fort III fracture
Tripod fracture.

Fractures to the temporal bone are particularly associated with extradural haematoma. Remember
the latent period with these injuries! Le fort III injuries describe a complex fracture which involves
the orbital floor, zygomatic arch and maxillary antrum. The mid portion of the face is basically
mobile and thus UNSTABLE.

Fractures to the paper-thin medial wall of the orbit (lamina papyracea) open the orbit and thus the
cranium to the ethmoid air cells (sinus). A tripod fracture often involves the orbital floor as well as
the lateral wall of the maxillary sinus, and a fluid level in the sinus represents blood and often
directs you to the area concerned.

141. Following an RTA, a 35-year-old woman, who had not been wearing a seat belt is brought into
A&E with a clear nasal discharge. Which type of fracture predisposes the patient to CSF
rhinorrhea?
Le Fort I fracture due to mobility of teeth YOUR ANSWER
Le Fort II due to fracture of the orbital floor
Le Fort III due to fracture of the ethmoid CORRECT ANSWER
Le Fort IV as it involves the frontal bone
Le Fort maxillary fractures are not associated with CSF rhinorrhea.

The French surgeon Le Fort performed experiments on cadavers in the early 1900s and classified
facial fractures into I, II and III. In Le Fort I injuries the fracture line passes above the palate,
fracturing the pyrimadal processes of the maxilla on each side, the vomer and the lower parts of
the pterygoid processes.

Bilateral Le Fort I fractures represent 23% of these injuries, unilateral 11%. Le Fort I and II fractures
occur together in 21% of cases. Le Fort III injuries involve the ethmoids and so increase
intercanthal distance. Nasopharyngeal intubation is preferred to oropharyngeal intubation to
secure the airway.

142. A 76-year-old man underwent tracheostomy for long term ventilation. A few weeks later the
tube was removed but the patient had hoarseness of voice. What is the most likely complication?
-------------------------------------------------------------------------------------------------------------------------------------------------------------------------------------------------
-------
Dr Mohammed Shamsul Islam Khan, Medical Officer, Clinical Neuro-Surgery, National Institute of Neuro-Sciences and Hospital
Sher-E-Bangla Nagar, Dhaka-1207, Bangladesh. Mobile: +880 1713 455 662, +880 1685 811979. E-mail: drsikhan@gmail.com
MyPasTest Online: MRCS A - Jan Exam 2015
01. Anatomy-Head, Neck & Spine (163Qs) 61

Surgical emphysema YOUR ANSWER


Pneumonia
Air embolism
Haemorrhage
Subglottic stenosis CORRECT ANSWER.
Page |
60
Other complications include pneumothorax, dislodgement of the tube (partial or complete),
obstruction of the tube or trachea, tracheal stenosis, sepsis, cuff prolapse, intubation granuloma of
the vocal cords. Subglottic stenosis occurs if the first and second tracheal rings are damaged.

143. A 32-year-old builder presents with back and severe left-leg pain radiating to his foot after
lifting a heavy box. MRI scan demonstrates an L5/S1 paracentral intervertebral disc bulge. With
these observations and findings which of the following is most likely to be correct? Select one
answer only.
An absent ankle jerk means that the disc should be removed YOUR ANSWER
Numbness on the sole of the foot may be found on examination CORRECT ANSWER
Pain radiating to the buttocks would be due to nerve root involvement
The exiting rather than the transiting nerve root is most likely to be compressed at this level
Urinary retention would never be secondary to pain and bedrest.

Nerve root involvement is suggested by pain radiating below the knee. It is worse when the nerve
is stretched, as with the sciatic stretch test, or when intra-abdominal pressure is raised, as with
straining at stool, coughing or sneezing. Urinary retention is a worrying sign but is not infrequently
secondary to pain, analgesic medication and bedrest. Before this can be assumed, a full
neurological examination must be performed, and if there are concerns regarding a caudaequina
syndrome then further investigations must be carried out.

Paracentral disc bulges are the most common in the lumbar region and usually impinge on the
transiting rather than the exiting nerve root. In this case that would be the S1 root which can result
in plantar flexion weakness, numbness on the sole of the foot in the S1 dermatome and an absent
ankle jerk reflex. An absent ankle jerkimplies some nerve root involvement, but on its own does not
demand surgical intervention.

144. One of your patients has a CT scan, which confirms a spinal tumour at the level of the
bifurcation of the trachea. An image-guided biopsy is to be performed. Which spinal level
corresponds to the bifurcation of the trachea?
Lower border C7 vertebra YOUR ANSWER
Upper border T2 vertebra
Lower border T2 vertebra
Upper border T3 vertebra
Lower border T4 vertebra CORRECT ANSWER.

-------------------------------------------------------------------------------------------------------------------------------------------------------------------------------------------------
-------
Dr Mohammed Shamsul Islam Khan, Medical Officer, Clinical Neuro-Surgery, National Institute of Neuro-Sciences and Hospital
Sher-E-Bangla Nagar, Dhaka-1207, Bangladesh. Mobile: +880 1713 455 662, +880 1685 811979. E-mail: drsikhan@gmail.com
MyPasTest Online: MRCS A - Jan Exam 2015
01. Anatomy-Head, Neck & Spine (163Qs) 61

The manubriosternal joint acts as a surface landmark for the tracheal bifurcation at T4. The cricoid
cartilage is at the C6 level, while the soft palate may reach down as far as C3.

The thyroid isthmus normally lies over the second and third tracheal rings and hence is a safe
place for an emergency surgical airway. The common carotid artery bifurcates into the external and
internal carotid arteries the upper border of the thyroid cartilage (C4).
Page |
145. A 19-year-old male is rushed to theatre after being stabbed in the neck during an alleged
assault. He is bleeding profusely and on arrival has a pulse of 130 and blood pressure of 76/40. In
61
theatre he is given 4 units of blood and surgical exploration shows a laceration to the external
carotid artery. At what level does the common carotid artery bifurcate into the external and internal
carotid arteries? Select one answer only.
Level of the cricoid cartilage YOUR ANSWER
Level of the hyoid bone
Level of the jugular notch
Lower border of the thyroid cartilage
Upper border of the thyroid cartilage CORRECT ANSWER.

The carotid artery divides into the external and internal carotid branches at the upper border of the
thyroid cartilage. The external carotid artery is deep to the hypoglossal nerve, having been crossed
by it at the level of the hyoid bone. The carotid sinus is normally found at the division of the
common carotid artery or at the commencement of the internal carotid artery.

The jugular or suprasternal notch is at T2 vertebral level, and as such is significantly inferior to the
bifurcation of the common carotid artery.

146. A 16-year-girl is brought in by her anxious parents as they have found a lump in her neck. The
lump is not painful and not causing any symptoms, on examination you note it is in the midline and
moves up when the child protrudes her tongue. Which of the following is the most likely cause for
this swelling?
Branchial cyst YOUR ANSWER
Dermoid cyst
Lymphoma
Sebaceous cyst
Thyroglossal cyst CORRECT ANSWER.

These occur if remnants of the thyroglossal duct persist. As this duct is a midline structure, they
occur in the midline and are more common in women mostly presenting between the ages of 15
and 30 years. On examination, they are 2-3cm in diameter, smooth and round and move up on
protrusion of the tongue.

147. Transection of the anterior division of the mandibular nerve (CN Vc) in the infra-temporal fossa
is most likely to result in which one of the following? Select one answer only.

-------------------------------------------------------------------------------------------------------------------------------------------------------------------------------------------------
-------
Dr Mohammed Shamsul Islam Khan, Medical Officer, Clinical Neuro-Surgery, National Institute of Neuro-Sciences and Hospital
Sher-E-Bangla Nagar, Dhaka-1207, Bangladesh. Mobile: +880 1713 455 662, +880 1685 811979. E-mail: drsikhan@gmail.com
MyPasTest Online: MRCS A - Jan Exam 2015
01. Anatomy-Head, Neck & Spine (163Qs) 61

Deviation of the jaw away from the side of the lesion on protrusion YOUR ANSWER
Dysphagia
Ipsilateral anaesthesia of the mandibular teeth
Ipsilateral anaesthesia of the mucosa of the oral vestibule CORRECT ANSWER
Ipsilateral paralysis of the buccinator muscle.
Page |
62
The anterior division of CN Vc has one sensory branch (the buccal nerve to the skin of the cheek
and mucosa of the vestibule). All other branches are motor to the muscles of mastication (the
masseter, temporalis and lateral pterygoid). The lower jaw teeth are supplied by the inferior
alveolar branch of the posterior division of CN Vc.

The buccinator muscle is supplied by the VIIth cranial nerve. Unopposed contraction of the
contralateral lateral pterygoid muscle deviates the jaw to the side of the lesion during protrusion.
None of the muscles of the pharynx are supplied by the anterior division of CN Vc, so dysphagia is
not a feature of damage to this nerve.

148. Whilst taking the anatomy viva section of the MRCS examination the examiner takes you to a
neck prosection. She then points to a tubular structure lying anterior to scalenus anterior, which
receives a branch from the retromandibular vein before piercing the deep cervical fascia and
joining the subclavian vein. What is this structure?
An anatomical variation, as no such structure has this anatomy YOUR ANSWER
The common carotid artery
The external carotid artery
The external jugular vein CORRECT ANSWER
The internal jugular vein.

The external jugular vein drains most of the scalp and side of the face. It begins near the angle of
the mandible and is formed from the union of retromandibular and postauricular veins, receiving
branches from the posterior external and transverse cervical veins. The external jugular vein has
two pairs of valves, which do not prevent regurgitation of the blood, or the passage of injection
from below upward.

The lower pair are placed at its entrance to the subclavian vein, the upper (in most cases) about
4cm above the clavicle. The external jugular vein lies anterior to scalenus anterior and pierces the
deep fascia of the neck, usually posterior to the clavicular head of the sternocleidomastoid muscle
before draining into the subclavian vein.

149. A 47-year-old female undergoes a thyroidectomy for Papillary carcinoma of the thyroid gland.
During this procedure the inferior thyroid arteries are identified and ligated lateral to the gland.
From which vessel does the inferior thyroid artery arise? Select one answer only.
Common Carotid artery YOUR ANSWER
External Carotid artery

-------------------------------------------------------------------------------------------------------------------------------------------------------------------------------------------------
-------
Dr Mohammed Shamsul Islam Khan, Medical Officer, Clinical Neuro-Surgery, National Institute of Neuro-Sciences and Hospital
Sher-E-Bangla Nagar, Dhaka-1207, Bangladesh. Mobile: +880 1713 455 662, +880 1685 811979. E-mail: drsikhan@gmail.com
MyPasTest Online: MRCS A - Jan Exam 2015
01. Anatomy-Head, Neck & Spine (163Qs) 61

Subclavian artery
Thyrocervical trunk CORRECT ANSWER
Thyroid ima artery.

The Superior thyroid artery is usually the first branch of the external carotid artery and descends to
the gland. In around 10% of people, a thyroid ima artery arises from the brachiocephalic trunk or
Page |
the arch of the aorta.The inferior thyroid artery is a branch of the thyrocervical trunk, which arises 63
from the subclavian artery.

The inferior thyroid artery supplies the inferior pole of the thyroid gland and gives off oesophageal
branches. In addition, it supplies both the superior and inferior parathyroid glands. The course of
the inferior thyroid artery is closely related to the recurrent laryngeal nerve: in a thyroidectomy,
therefore, it is preferable to ligate the artery lateral to the gland to avoid neural damage.

150. You are assisting in theatre, performing a median thoracotomy when the subclavian artery is
accidently damaged near its origin. Your foundation year doctor asks if you can simply tie off and
ligate the artery to stop the bleeding. What is the most appropriate answer?
It depends on which side. The artery can be sacrificed on the left but not the right YOUR ANSWER
No, the subclavian artery supplies the thyroid, breast and upper lobe of the lung
No, the subclavian artery supplies the thyroid gland, breast, rectus abdominus, brainstem and
diaphragm CORRECT ANSWER
No. Unless pre-operative angiography has been performed to verify the anatomy. In 50% of the
population the subclavian supplies the brain stem and in these people it cannot be sacrificed
Yes. The subclavian is a relatively minor artery and can be sacrificed with impunity.

The thyroid gland is supplied by the inferior thyroid artery, a branch of the thyocervical trunk. The
internal thoracic artery supplies: the breast, through anterior intercostal vessels, usually in the
second and third intercostal spaces; the rectus abdominis muscle, through the superior epigastric
branch; and the diaphragm, through the musculophrenic artery.

The vertebral arteries supply the brainstem through the posterior inferior cerebellar arteries.

151. A patient undergoes a radical parotidectomy for a malignant parotid tumour, at which time it is
found necessary to perform a total division of the left facial (VII) nerve at this level. Postoperatively,
which is the most likely sequel? Single best answer question choose ONE true option only.
Loss of left sided frown in all cases YOUR ANSWER
Numbness over the cheek on the left side
Ptosis of the upper eyelid on the left side
Loss of taste sensation over the anterior two-thirds of the tongue on the left side
Tendency for food and fluids to collect in the buccal sulcus after meals CORRECT ANSWER.

The facial nerve supplies all the muscles needed for facial expression including the
occipitofrontalis, which wrinkles the forehead. A distressing feature is paralysis of the buccinator
-------------------------------------------------------------------------------------------------------------------------------------------------------------------------------------------------
-------
Dr Mohammed Shamsul Islam Khan, Medical Officer, Clinical Neuro-Surgery, National Institute of Neuro-Sciences and Hospital
Sher-E-Bangla Nagar, Dhaka-1207, Bangladesh. Mobile: +880 1713 455 662, +880 1685 811979. E-mail: drsikhan@gmail.com
MyPasTest Online: MRCS A - Jan Exam 2015
01. Anatomy-Head, Neck & Spine (163Qs) 61

muscle, which acts to empty the buccal sulcus during mastication. There are no cutaneous
sensory fibres in the facial nerve.

The levator palpebrae superioris is supplied by the oculomotor nerve, so the patient can still raise
his upper lid. The chorda tympani fibres, which transmit taste from the anterior two-thirds of the
tongue, pass from the lingual nerve to the facial nerve just below the skull, and therefore remain
intact in peripheral injuries of the facial nerve. Page |
152. A 26-year-old female suffers with chronic submandibular gland infections and undergoes an
64
excision of the gland. Which of the following is a potential complication of surgery? Select one
answer only.
Anaesthesia of the contralateral half of the tongue YOUR ANSWER
Damage to the retromandibular artery
Deviation of the tongue to the contralateral side on protrusion
Freys syndrome
Weakness of the angle of the mouth CORRECT ANSWER.

The structures to be avoided at the time of submandibular gland excision include: the mandibular
branch of the facial nerve; the hypoglossal and lingual nerves; the facial artery and the
retromandibular vein. Facial nerve damage causes weakness of the angle of the mouth.

Lingual nerve damage results in ipsilateral anaesthesia and hemiplegia of the tongue. Hypoglossal
nerve injury restricts the mobility of the tongue with deviation to the ipsilateral side on protrusion.
Freys syndrome (gustatory sweating) is a complication of parotid surgery.

153. Skull fractures in the temporal region can damage the middle meningeal artery. Which of the
following best describes this artery?
It is a branch of the internal carotid artery YOUR ANSWER
It originates in the infratemporal fossa CORRECT ANSWER
It supplies the pia mater
It is contained within the arachnoid mater
It passes through the foramen ovale.

The middle meningeal artery is a branch of the maxillary artery which in turn is a branch of the
external carotid artery. It supplies the dura and calvaria but not the underlying arachnoid mater or
pia mater.

The middle meningeal artery lies on the bone outside the dura and passes through the foramen
spinosum. The foramen ovale transmits the accessory middle meningeal artery.

154. The anterior approach to the cervical spine is indicated to remove tumours and fuse the spine.
The platysma forms the most superficial plane. Which nerve supplies the platysma?
Facial nerve YOUR ANSWER (Correct)
-------------------------------------------------------------------------------------------------------------------------------------------------------------------------------------------------
-------
Dr Mohammed Shamsul Islam Khan, Medical Officer, Clinical Neuro-Surgery, National Institute of Neuro-Sciences and Hospital
Sher-E-Bangla Nagar, Dhaka-1207, Bangladesh. Mobile: +880 1713 455 662, +880 1685 811979. E-mail: drsikhan@gmail.com
MyPasTest Online: MRCS A - Jan Exam 2015
01. Anatomy-Head, Neck & Spine (163Qs) 61

Recurrent laryngeal nerve


Spinal accessory nerve
Sympathetic chain
Trigeminal nerve.
Page |
Seventy-five per cent of cases of acute facial nerve palsy are of unknown aetiology (Bells palsy);
hence computed tomography (CT) scanning is not usually required in the presence of few clinical 65
signs.

The HouseBrackmann scale of I (normal) to IV (complete paralysis) gives an indication of severity,


with eye closure the most important observation as this can lead to corneal ulceration (grade III
and above).

Marginal mandibular nerve damage in submandibular gland surgery manifests as drooping of the
corner of the mouth. Palsy following superficial parotidectomy is in the region of 0.5%.

155. Which of the following structures separates the anterior and posterior chambers in the
eyeball? Single best answer question choose ONE true option only.
The lens YOUR ANSWER
The cornea
The iris CORRECT ANSWER
The pupil
The ciliary processes.

The iris has received its name from its various colours in different individuals. It is a thin, circular,
contractile disc, suspended in the aqueous humour between the cornea and lens and perforated a
little to the nasal side of its centre by a circular aperture, the pupil. At its periphery it is continuous
with the ciliary body and is also connected to the posterior elastic lamina of the cornea by means
of the pectinate ligament.

Its surfaces are flattened and look forward and backward, the anterior toward the cornea, the
posterior toward the ciliary processes and lens. The iris divides the space between the lens and the
cornea into an anterior and a posterior chamber. The anterior chamber of the eye is bounded in
front by the posterior surface of the cornea; behind by the front of the iris and the central part of
the lens.

The posterior chamber is a narrow chink behind the peripheral part of the iris and in front of the
suspensory ligament of the lens and the ciliary processes. In the adult, the two chambers
communicate through the pupil, but in the fetus up to the seventh month they are separated by the
membrana pupillaris.

156. A 25-year-old male presents with significant facial trauma following a road traffic collision with
proptosis of the right eye. CT scan demonstrates a fracture across the superior orbital fissure with
associated haematoma. Which nerve is least likely to be affected? Select one answer only.
The frontal nerve YOUR ANSWER

-------------------------------------------------------------------------------------------------------------------------------------------------------------------------------------------------
-------
Dr Mohammed Shamsul Islam Khan, Medical Officer, Clinical Neuro-Surgery, National Institute of Neuro-Sciences and Hospital
Sher-E-Bangla Nagar, Dhaka-1207, Bangladesh. Mobile: +880 1713 455 662, +880 1685 811979. E-mail: drsikhan@gmail.com
MyPasTest Online: MRCS A - Jan Exam 2015
01. Anatomy-Head, Neck & Spine (163Qs) 61

The inferior division of the oculomotor nerve


The lacrimal nerve
The nasociliary nerve
The zygomatic nerve CORRECT ANSWER.
Page |
The superior orbital fissure lies at the apex of the orbit bounded primarily by the greater and lesser
wings of the sphenoid bone. The tendons of the lateral rectus divide the fissure into superior and 66
inferior parts. The trochlear, frontal and lacrimal nerves pass superiorly with the superior orbital
vein. The superior and inferior branches of the oculomotor nerve, abducens and nasociliary nerves
pass inferiorly with the inferior orbital vein.

The frontal and nasociliary nerves arise from the ophthalmic division of the trigeminal nerve in the
lateral wall of the cavernous sinus. After exiting the superior orbital fissure, the frontal nerve
divides into the supratrochlear and supraorbital nerves at the orbital margin to supply the skin of
the forehead.

The nasociliary nerve supplies the ethmoidal sinuses, sphenoidal sinuses, skin of the upper
eyelids and nose. The inferior ramus of the oculomotor nerve gives off branches to the inferior
rectus, medial rectus and the inferior oblique muscles. The superior ramus of the oculomotor
nerves supplies the levatorpalpebraesuperioris and so may give rise to a ptosis if cut.

The zygomatic nerve is a branch of the maxillary division of the trigeminal nerve which enters the
orbit through the inferior orbital fissure.

157. A 62-year-old heavy smoker is noted by his dentist to have a white patch on his oral mucosa
during routine assessment. This cannot be scraped off easily. What is the most likely cause for this
lesion?
Candidiasis YOUR ANSWER
Erythroplasia
Leucoplakia CORRECT ANSWER
Lichen planus
Tertiary syphilis.

All of the above lesions are pre-malignant. High-risk lesions are those that in 25% of cases will
become malignant in 5 years. The lesions in leukoplakia cannot be scraped off. High-risk lesions
are rare (especially syphilis) but are important to bear in mind when patients present with lesions
in the mouth.

158. A 67-year-old male suffers an embolic stroke. Duplex ultrasonography demonstrates a severe
right common carotid artery stenosis and he proceeds to undergo a carotid endarterectomy. Which
of the following best describes the right common carotid artery? Select one answer only.
It bifurcates at the level of the upper border of the cricoid cartilage YOUR ANSWER
It has the cervical sympathetic chain as an anterior relation
It is a branch of the aortic arch
-------------------------------------------------------------------------------------------------------------------------------------------------------------------------------------------------
-------
Dr Mohammed Shamsul Islam Khan, Medical Officer, Clinical Neuro-Surgery, National Institute of Neuro-Sciences and Hospital
Sher-E-Bangla Nagar, Dhaka-1207, Bangladesh. Mobile: +880 1713 455 662, +880 1685 811979. E-mail: drsikhan@gmail.com
MyPasTest Online: MRCS A - Jan Exam 2015
01. Anatomy-Head, Neck & Spine (163Qs) 61

It is enclosed within the carotid sheath throughout its course CORRECT ANSWER
It lies lateral to the lateral lobe of the thyroid gland.

The right common carotid artery branches off the brachiocephalic artery. It bifurcates at the level of
the upper border of the lamina of the thyroid cartilage.
Page |
It lies posterior to the lobes of the thyroid gland and anterior to both the cervical sympathetic chain 67
and the phrenic nerve on the scalenus anterior muscle; the latter is separated from the artery by
prevertebral fascia.

159. A 45-year-old woman is referred to the neurosurgical outpatient clinic by her GP after she
complained of gradual loss of hearing in her left ear, with associated tinnitus and facial weakness.
What is the most likely site of the lesion?
The foramen lacerum YOUR ANSWER
The foramen magnum
The foramen rotundum
The foramen spinosum
The internal auditory canal CORRECT ANSWER.

This patient has presented with symptoms consistent with a vestibular schwannoma also known
as an acoustic neuroma. These are benign primary intracranial tumours arising from the myelin
forming cells of the vestibulo-cochlear (CN V111) nerve. The foramen magnum transmits the
vertebral arteries, which unite at the lower border of the pons to form the basilar artery.

The foramen spinosum transmits the middle meningeal vessels and the meningeal branch of the
mandibular nerve. The foramen rotundum contains the maxillary nerve. The foramen ovale
transmits the mandibular nerve, lesser petrosal nerve and accessory meningeal artery. The
foramen lacerum transmits the internal carotid and greater petrosal nerve, which leaves as a nerve
of the pterygoid canal.

160. A 29-year-old female underwent total thyroidectomy. Post-operatively she complained of


difficulty in breathing. What would be the most likely cause of this?
Unilateral recurrent laryngeal nerve damage during dissection of the upper pole vessels YOUR
ANSWER
Hypoparathyroidism in 10% cases
Laryngeal oedema CORRECT ANSWER
Tachycardia, hyperpyrexia and confusion
Damage to the external superior laryngeal nerve.

A thyroid crisis should be suspected if there is tachycardia, hyperpyrexia and confusion. It is a


potentially life-threatening condition that is rarely seen but classically occurs in a patient who has
-------------------------------------------------------------------------------------------------------------------------------------------------------------------------------------------------
-------
Dr Mohammed Shamsul Islam Khan, Medical Officer, Clinical Neuro-Surgery, National Institute of Neuro-Sciences and Hospital
Sher-E-Bangla Nagar, Dhaka-1207, Bangladesh. Mobile: +880 1713 455 662, +880 1685 811979. E-mail: drsikhan@gmail.com
MyPasTest Online: MRCS A - Jan Exam 2015
01. Anatomy-Head, Neck & Spine (163Qs) 61

undergone surgery without adequate preoperative preparation. Assessment of damage to the


external superior laryngeal nerve is difficult, as changes are subtle.

It is most likely to be damaged at the time of ligation and division of the superior thyroid vessels.
To avoid this, arterial branches should be ligated individually and close to the thyroid and the
nerve identified whenever possible. Persistent hypoparathyroidism should be less than 1% in Page |
reputable units.
68
161. You are reviewing a patient with a facial nerve palsy after resection of an acoustic neuroma.
The facial nerve supplies the:
Buccinator muscle YOUR ANSWER (Correct)
Medial pterygoid muscle
Parasympathetics to the parotid gland
Sensation to the inside of the mouth
Sympathetics to the lacrimal gland.

The facial nerve gives off a branch to the stapedius before it enters the stylomastoid foramen. The
buccinator is supplied by the buccal branches of the facial nerve. The medial (and lateral)
pterygoid is supplied by the mandibular division of the trigeminal nerve.

The parasympathetic supply to the lacrimal gland is from the secretomotor fibres from the superior
salivatory nucleus travelling in the intermediate and greater petrosal nerves and relaying in the
pterygopalatine ganglion.

The parasympathetic supply to the parotid gland is from the inferior salivatory nucleus via the
tympanic and lesser petrosal branches of the glossopharyngeal nerve that project to the otic
ganglion (preganglionic).

Postganglionic fibres pass from the otic ganglion to the parotid via the auriculotemporal nerve.

162. Which of the following extraocular muscles is most important for eye abduction? Single best
answer question choose ONE true option only.
Lateral rectus muscle YOUR ANSWER (Correct)
Inferior rectus muscle
Superior oblique muscle
Inferior oblique muscle
Medial rectus muscle.

The lateral rectus muscle is an extra-ocular muscle that serves primarily to abduct the eye from the
midline. It arises from the lateral part of the tendinous ring within the posterior orbit, and inserts
into the lateral sclera of the eye anterior to the equator but posterior to the sclerocorneal junction.
The lateral rectus muscle is innervated by the abducens nerve (CN VI).

-------------------------------------------------------------------------------------------------------------------------------------------------------------------------------------------------
-------
Dr Mohammed Shamsul Islam Khan, Medical Officer, Clinical Neuro-Surgery, National Institute of Neuro-Sciences and Hospital
Sher-E-Bangla Nagar, Dhaka-1207, Bangladesh. Mobile: +880 1713 455 662, +880 1685 811979. E-mail: drsikhan@gmail.com
MyPasTest Online: MRCS A - Jan Exam 2015
01. Anatomy-Head, Neck & Spine (163Qs) 61

163. A 33-year-old male presents to his GP concerned about several 1mm- white spots around the
vermillion border of his lips. On examination they are smooth and non-tender. Which one of the
following is most likely the cause?
Bowens disease YOUR ANSWER
Candidiasis
Fordyces granules CORRECT ANSWER
Page |
Hairy leucoplakia 69
Speckled leucoplakia.

Fordyces granules are creamy spots of sebaceous glands and are frequently mistaken for disease.
Oral candida has a low malignant potential. True frictional keratosis has no malignant potential.
Hairy leucoplakia strongly suggests HIV infection; if negative other rarer immunodeficiency
syndromes should be considered.

Editor:
Dr Mohammed Shamsul Islam Khan
MBBS (CMC); FCPS-II (Neuro-Surgery)
Medical Officer, Clinical Neuro-Surgery
National Institute of Neuro-Sciences and Hospital
Sher-E-Bangla Nagar, Dhaka-1207, Bangladesh.
Mobile: +880 1713 455 662, +880 1685 811979.
E-mail: drsikhan@gmail.com

-------------------------------------------------------------------------------------------------------------------------------------------------------------------------------------------------
-------
Dr Mohammed Shamsul Islam Khan, Medical Officer, Clinical Neuro-Surgery, National Institute of Neuro-Sciences and Hospital
Sher-E-Bangla Nagar, Dhaka-1207, Bangladesh. Mobile: +880 1713 455 662, +880 1685 811979. E-mail: drsikhan@gmail.com

Anda mungkin juga menyukai